Nothing Special   »   [go: up one dir, main page]

FAR Answers

Download as pdf or txt
Download as pdf or txt
You are on page 1of 67

90 | THE FINANCIAL REPORTING ENVIRONMENT

ANSWERS TO QUICK REVISION QUESTIONS 1

1 D The primary aim of accounting is to provide financial information for users.


2 C Lenders and shareholders in particular are identified as the primary users of financial
statements. Information produced with them in mind should also be useful for other user
groups, however these other groups are not considered to be the main audience of financial
accounts.
3 D Although shareholders need to know the future prospects, they also need to know that the
current position of the company is secure. Similarly, suppliers need to know the future
prospects to ensure that they will be paid.
4 C The IASB is overseen by and accountable to the IFRS Foundation. The IASB is responsible for
issuing IFRSs.
5 D A, B and C are not correct as the IFRS Foundation appoints the members of the IASB; the IFRS
Interpretations Committee issues Interpretations; and the IFRS Foundation oversees the work of
the IFRS Interpretations Committee. D is correct as the IFRS Advisory Council assists and
advises the IASB in the process of developing IFRSs.
6 B The IASB has no powers of enforcement.
7 C The IFRS Interpretations Committee interprets the application of IFRS and provides guidance
on topics not specifically covered by an IFRS.
8 D GAAP is all the rules and regulations a company must follow so can include national as well as
international standards.
9 B A, C and D – under the regulated framework, financial statements of the company will result in:
more disclosure being required; the extra work involved in adhering to regulations is costly and
competitors have access to more information which they may use to their advantage.
Regulation does, however, result in higher quality, more comparable, relevant and faithfully
represented information.
10 D Although the issue of a discussion paper is not a mandatory step in due process, this would be
issued before the (mandatory) exposure draft, and in due course a final standard.

ANSWERS TO QUICK REVISION QUESTIONS 2

1 D A conceptual framework is sometimes referred to as a 'guiding light' which underpins


accounting standards.
2 A The framework provides the principles which underpin IFRS; in addition its principles are
applied where no standard exists. Therefore all transactions are effectively accounted for in line
with the framework, so resulting in standardised accounting practice.
3 B The Conceptual Framework for Financial Reporting is the name of the IASB's conceptual
framework.
4 B All are valid reasons that financial statements are produced, but the key reason is B.
5 A Financial statements are not used to value a company, methods such as historic cost mean
valuations do not necessarily represent the current value of the entity.
6 A Existing and potential investors, lenders and other creditors (providers of capital).
7 B The requirements of IFRSs dealing with similar and related issues. Whilst guidance should also
be sought from the underlying principles of the Conceptual Framework, this is secondary to
consulting similar IFRSs.
8 B A is a change of accounting estimate, C is specifically mentioned in IAS 8 as not constituting a
change of accounting policy, D is a change in an estimate.
9 B Material errors are treated in the same way as changes of accounting policy. They are corrected
retrospectively, so that the financial statements are presented as if the error had never
occurred.
FINANCIAL ACCOUNTING AND REPORTING | 91

10 B All of the other options are changes in accounting estimates. A change in accounting estimate
is defined by IAS 8 as ‘an adjustment of the carrying amount of an asset or a liability, or the
amount of the periodic consumption of an asset, that results from the assessment of the present
status of, and expected future benefits and obligations associated with, assets and liabilities’. If
company changes its inventory valuation method to FIFO from weighted average method then
it is basically changing the principle of valuation as FIFO follows a particular cost flow
assumption whereas weighted average method uses weighted average of the cost at which

MODULE 1
inventory was held at the beginning of the period and cost of the goods bought during the
period. So, both methods use different basis to value the closing inventory this is a change in
measurement basis. Therefore, it is a change in accounting policy.
Under IAS 8, accounting policies are defined as ‘the specific principles, bases, conventions,
rules and practices applied by an entity in preparing and presenting financial statements’.
Presenting incorporates which heading an item is reported under in a primary statement which
is why B is a change in accounting policy.
11 C Decrease in the opening value of inventory would be considered to be a change of accounting
estimate, so no prior period adjustment would be required.
Discovery of significant fraud would be classed as a material error, however the error only
relates to this period. Therefore, a prior period adjustment is not necessary. Similarly, the wrong
tax charge would also be a material error, if it was based on incorrect information, this would
require a prior period adjustment.
The change in method of calculating irrecoverable debt provision would be a change in
accounting estimate. Therefore no prior period adjustment would be required.
92 | THE FINANCIAL REPORTING ENVIRONMENT

ANSWERS TO QUICK REVISION QUESTIONS 3

1 B A principles based system discourages 'creative accounting' abuses. Principles are harder to
evade than rules. Many people believe that A and C are advantages of a rules based system. D
is often true of principles based standards, but many view this as a disadvantage.
2 C Application of IFRS will not result in completely accurate financial statement, due to the use of
estimates and judgements.
3 B The other arguments are all in favour of accounting standards.
4 C The going concern basis should be used unless an entity has the intention or need to enter
liquidation or cease trading. If an entity has concerns or doubts the going concern basis, but
made the assessment that the going concern basis is appropriate, then the going concern basis
applies and supported by additional disclosures.
5 B These are the four qualitative characteristics contained within the Conceptual Framework which
enhance the usefulness of information that is relevant and faithfully represented.
6 C Materiality concerns whether omitting or misstating an item of information in the financial
statements could influence decisions that the primary users of general purpose financial reports
make on the basis of those reports.
A faithful representation must be free from error, but this does not mean perfectly accurate in
all respects.
Faithful representation requires an item to be complete, neutral and free from error.
7 A The accruals concept requires that the effects of transactions are recognised when they occur,
so meaning that credit sales and purchases, for example, are included in profit or loss for a
period.
8 A 17 IFRS have been published by the IASB.
9 A Investors will benefit as financial statements will be more comparable.
10 C The FASB is the US standard setter. It has worked with the IASB on a number of projects,
including new standards on business combinations, fair value measurement, financial
instruments and revenue recognition.

ANSWERS TO QUICK REVISION QUESTIONS 4

1 A This is the definition of an asset contained within the Conceptual Framework and various IFRS.
2 D This is the definition of a liability contained within the Conceptual Framework. A is the definition
of equity; C is the definition of an asset; B for an obligation to meet the definition of a liability, it
does not need to be certain, or even likely, that the entity will be required to transfer an
economic resource, the obligation must just have the potential to require a transfer of
economic resource.
3 D For an item to recognised in the financial statements it must meet the definition of an element
recognition must provide relevant information and a faithful representation of the
transactions of the entity.
4 C Assets, liability and equity are included in the statement of financial position.
5 D Motor vehicles are generally a non-current asset, however motor vehicles held for sale as part of
a trade are current assets in accordance with IAS 2 Inventories. Property, plant and equipment
and licences are non-current assets of a business; retained earnings are part of the equity in a
business.
6 B A bank loan is a liability of a business and inventory is a current asset.
7 B An overdraft is classed as current, even where there is a rolling facility, as it is repayable on
demand.
FINANCIAL ACCOUNTING AND REPORTING | 93

8 C Payable accounts are part of the normal operating cycle of a business, and as such are classified
as current liabilities, even where the credit period exceeds 12 months.
9 A B, C and D are all assets.

ANSWERS TO MODULE QUESTIONS

MODULE 1
1 B Financial reporting is carried out by all businesses, no matter what their size or structure.
2 A Customers need to know that the business is making sufficient profits to be a secure source of
supply.
3 Other examples of areas where the judgment of different people may vary are as follows:
(a) valuation of buildings in times of rising property prices
(b) research and development: is it right to treat this only as an expense? In a sense it is an
investment to generate future revenue.
(c) accounting for inflation
(d) brands such as 'Coca Cola' or 'Hoover' – are they assets in the same way that a fork lift truck is
an asset?
Working from the same data, different groups of people may produce very different financial
statements. If the exercise of judgment is completely unrestrained, there will be no comparability
between the accounts of different organisations. This will be all the more significant in cases where
deliberate manipulation occurs, in order to present accounts in the most favourable light.
4 Methods of 'creative accounting' include:
 'Off balance sheet financing': an entity enters into a financing transaction which is structured
so that it can avoid having to recognise all its assets and liabilities in the statement of financial
position. For example, a company might sell an asset but enter into an agreement to
repurchase it after a set period of time. The substance of the transaction is that the company
has a loan (a liability) secured on the asset that has been 'sold' but legally, the company has
made a sale and so recognises cash and income. The company's financial performance and
particularly its financial position appear to be much stronger than they are. Transactions such as
these enable a company to 'hide' material borrowings from shareholders and other lenders.
 'Window dressing': at the year-end an entity enters into transactions whose sole purpose is to
improve the appearance of the financial statements. For example, a company might make a
fictitious 'sale', which would be reversed by means of a credit note early in the new reporting
period. Revenue and profit would appear to be higher than they really were.
 'Profit smoothing': in a profitable year an entity deliberately recognises a liability for future
expenditure to which it is not committed (for example, for a 'restructuring' or for future losses).
This 'provision' is then available to be released to profit or loss to increase profits in a poor year
(the provision is sometimes called the 'big bath').
 'Aggressive earnings management': recognising sales revenue before it has been earned
(before the entity has actually delivered the goods or performed the services).
Most of the accounting scandals of the past 20 years have involved one or more of these. For
example, the management of Enron used a sophisticated form of off balance sheet financing to
mislead the users of its financial statements.
5 The types of economic decisions for which financial statements are likely to be used include the
following:
 decisions to buy, hold or sell equity investments;
 assessment of management stewardship and accountability;
 assessment of the entity's ability to pay employees;
 assessment of the security of amounts lent to the entity;
 determination of taxation policies;
94 | THE FINANCIAL REPORTING ENVIRONMENT

 determination of distributable profits and dividends;


 inclusion in national income statistics; and/or
 regulations of the activities of entities.
6 The nine user groups who are likely to be interested in financial information about a company are:
 Managers of the company will need information about the company's financial situation as it is
currently and as it is expected to be in the future. This is to enable them to manage the
business efficiently and to make effective decisions.
 Shareholders of the company will want to assess how well the management is performing.
They want to know how profitable the company's operations are and how much profit they can
afford to withdraw from the business for their own use.
 Trade contacts include suppliers who provide goods to the company on credit and customers who
purchase the goods or services provided by the company. Suppliers want to know about the
company's ability to pay its debts; customers need to know that the company is a secure source of
supply and is in no danger of having to close down.
 Providers of finance to the company will wants to ensure that the company is able to keep up
interest payments, and eventually to repay the amounts advanced.
 The taxation authorities want to know about business profits in order to assess the tax payable
by the company, including sales taxes, for example, Goods and Services Tax or Value Added
Tax.
 Employees of the company should have a right to information about the company's financial
situation, because their future careers and the size of their wages and salaries depend on it.
 Financial analysts and advisers need information for their clients or audience. As examples,
stockbrokers need information to advise investors; credit agencies want information to advise
potential suppliers of goods to the company; and journalists need information for their reading
public.
 Government and their agencies are interested in the allocation of resources and therefore in
the activities of business entities. They also require information in order to provide a basis for
national statistics.
 The public. Companies affect members of the public in a variety of ways. They may make a
substantial contribution to a local economy by providing employment and using local suppliers.
Another important factor is the effect of an entity on the environment as an example in relation
to pollution.
7 (a) This is a change in presentation, so it does represent a change of accounting policy.
(b) This is a change of accounting estimate, not a change of accounting policy.
(c) This is a change of measurement basis, so it does represent a change of accounting policy.
8 STATEMENT OF PROFIT OR LOSS
20X7 20X6
$'000 $'000
Sales 54 000 50 000
Cost of goods sold (25 930) (24 050)
Profit before tax 28 070 25 950
Income tax (8 421) (7 785)
Profit for the year 19 649 18 165

9 Change in accounting estimate


The change in accounting estimate affects the accounts for 20X7 and subsequent years. At
31 December 20X7 the NBV of the asset is $60 000 (assuming that 20X7 depreciation has not yet
been charged). This should be spread over the revised remaining useful life, that is, three years.
Depreciation on the asset for 20X7, 20X8 and 20X9 will be $20 000 per annum.
FINANCIAL ACCOUNTING AND REPORTING | 95

10 STATEMENT OF PROFIT OR LOSS AND OTHER COMPREHENSIVE INCOME


20X7 20X6
$'000 $'000
Sales 67 200 47 400
Cost of goods sold (W1) (51 600) (38 770)
Profit before tax 15 600 8 630

MODULE 1
Income tax (W2) (4 680) (2 589)
Profit for the year 10 920 6 041
RETAINED EARNINGS
20X7 20X6
Opening retained earnings $'000 $'000
As previously reported 21 981 13 000
Correction of prior period
error (4200 – 1260) (2 940) –
As restated 19 041 13 000
Profit for the year 10 920 6 041
Closing retained earnings 29 961 19 041

Workings
1 Cost of goods sold 20X7 20X6
$'000 $'000
As stated in question 55 800 34 570
Inventory adjustment (4 200) 4 200
51 600 38 770
2 Income tax 20X7 20X6
$'000 $'000
As stated in question 3 420 3 849
Inventory adjustment (4200 × 30%) 1 260 (1 260)
4 680 2 589

11 (a) IAS 38 is generally viewed as a principles-based standard. This is despite that there is a minor
component of rules involved, such as 'research must be expensed and development costs are
capitalised if all of the conditions are met'. However, this rule is being overruled by the principle
approach, that is, by requiring management to assess whether they meet the criteria.
(b) There is a danger that management might ignore the basic principle and simply apply the rules,
particularly if this improves the entity's financial position. They might decide to capitalise all
development costs rather than making the judgements required. However, given IAS 38 is a
principles-based standard, it is harder for management to ignore the principles as the standard
requires management to assess whether they meet the criteria for recognition, not just
capitalise on development costs that do not meet the criteria.
12 (a) If the business is to be closed down, the remaining three machines must be valued at the
amount they will realise in a forced sale, that is, 3 × $60 = $180.
(b) If the business is viewed as a going concern, the inventory unsold at 31 December will be
carried forward into the following year, when the cost of the three machines will be matched
against the eventual sale proceeds in computing that year's profits. The three machines will
therefore be valued at cost, 3 × $100 = $300.
13 (a) This is an intangible asset. There is a past event, control and future economic benefit as a result
of cost savings.
(b) This cannot be classified as an asset. Baldwin Co. has no control over the car repair shop and it
is difficult to argue that there are 'future economic benefits'.
(c) The bonus is a liability as the business has created an obligation.
14 The accounting policies apply the requirements of IAS 38 Intangible Assets, but in some aspects
conflict with the definitions and the recognition criteria in the Conceptual Framework for Financial
Reporting.
96 | THE FINANCIAL REPORTING ENVIRONMENT

An intangible asset is recognised under IAS 38 if:


• it is probable that the expected future economic benefits attributed to the asset will flow to the
entity; and
• the cost of the asset can be measured reliably
Although expenditure on research activities may eventually result in future economic benefits (and
therefore there may be an asset) it cannot be capitalised because it does not meet the recognition
criteria: it is too early to say whether there will actually be any economic benefits or to be able to
make any kind of reliable estimate of the amount.
In contrast, development expenditure is capitalised if it meets certain criteria. There is an asset: the
new product and the ideas behind it are controlled by the entity and there is expected to be an
inflow of economic benefits in the form of increased revenue or reduced costs. An intangible asset
is recognised if the product or process is technically and commercially feasible and there are
sufficient resources to complete development. If these criteria are met the inflow of economic
benefits is probable. The second criteria of reliable measurement is also met because the
amount to be capitalised is the cost of materials, labour and a proportion of overheads; these
amounts will be recorded in the company's accounting system.
If development expenditure does not meet the criteria it is not recognised as an asset, but as an
expense in the period in which it is incurred.
The definition of an asset under the Conceptual Framework does not require the probable inflow
of future economic benefit, only the potential to produce economic benefit, these benefits do not
need to be certain or even likely. In most cases it is likely that research and development would be
carried out with some type of economic benefit in mind. Therefore the definition of an asset
according to the Conceptual Framework conflicts with that of IAS 38.
The Conceptual Framework also does not specify a reliable measurement, instead it requires the
information to be relevant and a faithful representation. The unknown outcome of the research and
development could make it uncertain whether an asset exists, there seems to be much more
certainty around the development costs that are technically feasible, whereas the research and
other development costs are very uncertain, therefore treating these as an asset may not give
relevant information.
The Conceptual Framework itself is not an International Financial Reporting Standard and so does
not overrule any individual IFRS. In the rare cases of conflict between an IFRS and the Conceptual
Framework, the IFRS will prevail.
15 Refunds
Cloud 11 should recognise a liability for the best estimate of costs of refunds on goods sold to
customers prior to the year end based on the following analysis.
A liability is a present obligation to transfer an economic resource as a result of past events.
Considering the three aspects of this definition:
• Cloud 11 has a constructive obligation to give dissatisfied customers refunds for returned
goods. This is because through its published policy (on its website, in its advertising and on
signs in its store), Cloud 11 has created a valid expectation in its customers that it will refund all
returned goods.
• The obligation has the potential to require Cloud 11 to transfer economic resource in the form
of a refund to customers if they return unwanted goods.
• A past event has occurred because Cloud 11 has taken an action in the form of sales to its
customers and, as a consequence, it may have to transfer an economic resource in the form of a
refund that otherwise it would not have had to.
The information would be relevant as it is very likely that some customers will seek refunds,
therefore the probability that there would be a transfer of economic benefits is not low. As the
policy has been in place for several years it is likely that a reliable estimate can be made based on
previous refund levels.
FINANCIAL ACCOUNTING AND REPORTING | 97

16 The preference shares are a non-current liability and should not be presented as part of equity in
the statement of financial position. They have the characteristics of a loan, rather than an owners'
interest. A liability exists because the company has an obligation to pay interest over the life of the
'shares' and eventually to repay the principal amount.
17 In order for an entity to recognise revenue, the conceptual framework states the entity must have
the right and potential to produce economic benefits and be able to exercise control. Customers

MODULE 1
pay for airline tickets before they actually receive the service that they have paid for. The terms of
airline tickets vary. In some cases the passenger can only fly on the date and to the destination
originally booked, but in other cases tickets may be exchangeable, transferable or refundable or
they may be valid for travel during a particular period, rather than on a specific flight. So whilst the
company does have an increase in assets (the cash it receives, which has the potential to produce
economic benefit) it also has an obligation, in that it has an obligation to deliver a service (the
flight), or even the possibility of a repayment to the customer, therefore the net assets have not
increased.
The airline group does not recognise revenue until passengers actually travel, that is, when it
actually delivers the service that has been paid for. Depending on the terms of the ticket, until that
time the company probably has a liability in the form of an obligation to make a refund to the
customer or to offer another flight. When the customer actually travels, the liability is discharged.
The recognition conditions are met: there is a decrease in a liability, and a certain inflow of
economic benefit which can be reliably measured.
Unused tickets can be recognised as revenue in certain conditions. For example, where a customer
books a ticket that only permits travel on a specific flight and then fails to travel, the company still
receives the cash paid for the ticket (an inflow of economic benefit that meets both recognition
conditions) but has no obligation to provide another flight.
18 In the current period
The company has sunk a mine in the current period but has not yet commenced the extraction of
minerals.
As a result of sinking the mine, at the year-end, the company has a legal obligation to restore the
site at the end of the mine's operating life. It therefore has a liability which meets both the
recognition criteria:
(a) A present obligation to transfer an economic resource as a result of past events; and
(b) It is relevant and will provide users of the financial statements with a faithful representation of
the transactions of the entity. The amount can be reliably estimated (on the basis of the amount
it would cost to restore the site at the year-end, adjusted as necessary for expected future
changes in technology etc).
The company should therefore recognise a liability in respect of 40 per cent of the total cost of
restoring the site at the end of the mine's operating life. The remaining 60 per cent of the eventual
cost of restoration is the result of extracting minerals. As this activity has not yet commenced, there
is no related liability.
The restoration costs recognised as a liability also meet the definition of an asset (the expenditure
will generate future economic benefits in the form of sales revenue) and also meet the recognition
criteria: an inflow of economic benefit is probable and the cost of restoration can be reliably
estimated. Therefore these costs form part of the cost of the mine within non-current assets.
In subsequent periods
The remaining 60 per cent of the eventual total cost of restoration relates to damage caused
progressively as minerals are extracted. Therefore as this damage occurs over the mine's operating
life, the liability for restoration costs is increased.
98 | THE FINANCIAL REPORTING ENVIRONMENT

19 B A complete set of financial statements for a limited company (reporting entity) normally
includes:
 a statement of the entity's financial position;
 a statement or statements showing the entity's financial performance;
 a statement showing changes in the entity's financial position (usually a statement of cash
flows);
 a statement showing changes in equity; and
 notes to the financial statements and other supplementary information.
144 | THE ACCOUNTING THEORY

Module 2
ANSWERS TO QUICK REVISION QUESTIONS 1
1 B Under the historical cost convention, assets are recorded at the amount of cash or cash
equivalents paid or the fair value of the consideration given to acquire them at the time of their
acquisition.
2 A Historical cost is the measurement basis most widely applied in financial statements.
3 C Assets will tend to be understated and profits overstated due to low depreciation charges.
4 C (($8 000  7) + ($4 000 – $1 000)) = $59 000.
5 C Cost of new machine and upgrade part ($14 000 + $2 500)  2/6 = $5 500
6 D Positive accounting theory is based on actual accounting practice. Normative accounting theory
explains what should occur rather than predicting what actually does occur.
7 C Capital maintenance. An entity has maintained its capital if the net assets total at the end of the
period is the same as that at the beginning of the period. If capital (assets less liabilities) is
greater at the end of the period than at the beginning, the entity has made a profit.
8 D Specific price inflation measures price changes over time for a specific group of assets. General
price inflation is the average rate of inflation that reduces the general purchasing power of
money.
9 A Statement III refers to current cost accounting. Statement I refers to historical cost accounting.
10 A Deprival value is the lower of replacement cost and recoverable value. Recoverable value is the
higher of sales value (net realisable value) and value in use (economic value).
11 C It avoids the overstatement of profit which can arise during periods of inflation. Historical cost
accounting does not avoid the overstatement of profit which arises during periods of inflation,
which is why alternative models have been proposed.
12 C In the case of C, what tends to happen when profits are overstated is that too much cash is paid
out in dividends to shareholders, depleting funds needed for investment. It is a widely held
principle that distributable profits should only be recognised after full allowance has been made
for any erosion in the capital value of a business. In historical cost accounts, although capital is
maintained in nominal money terms, it may not be in real terms. So, inflated profits may be
distributed to the detriment of the long-term viability of the business.
13 B $320 000 Historical cost; $384 000 current cost
Historical cost Current cost
$'000 $'000
Cost/valuation 500 600
Depreciation ((500 000 × 90%) / 5) × 2 (180)
Depreciation ((600 000 × 90%) / 5) × 2 (216)
Carrying amount 320 384

14 A Current cost accounting


FINANCIAL ACCOUNTING AND REPORTING | 145

ANSWERS TO QUICK REVISION QUESTIONS 2

1 B An agency relationship involves a contract under which the principals engage the agent to
perform some service on their behalf and delegate some decision-making authority to the
agent.
2 D Company directors owe a fiduciary duty to a company to exercise their powers in what they
honestly consider to be the interests of the company. This duty is owed to the company and not
generally to individual shareholders.
3 C Agency costs include:
 salaries paid to directors in their role as agents
 monitoring costs to ensure that the agent is doing what they should be (such as the costs of
preparing financial information, the cost of an external auditor and the internal audit
function).
4 B Other mandatory items include the statement of financial position, statement of cash flows and

MODULE 2
statement of changes in equity. Narrative reports such as a risk review and environmental report
are not mandatory, however these may be included in the annual report.
5 B The corporate governance report is required as part of the listing rules. The Chairman's
Statement is voluntary.
6 A Per IAS 1, accounting policies must be disclosed. Provision of a corporate governance
statement would be good practice but as the company is unlisted it is not mandatory.
7 C The board of directors. They may delegate their responsibility to the finance department but
they will still retain the overall responsibility.
8 B Paying management element of profit related pay can help alleviate the agency problem as it
incentivises management to work on behalf of the company, therefore item I is incorrect.
However incentive schemes can also lead to creative accounting and manipulation of financial
statements if mangers are reliant on results to earn salary/bonus.
9 C Disclaimer.
10 A A management commentary should include the management’s view of the business, it should
also have some elements that are forward looking, therefore statement 2 is incorrect.
11 B The directors' report is part of the other information included in a financial report, it is not part
of the main financial statements themselves.
146 | THE ACCOUNTING THEORY

ANSWERS TO MODULE QUESTIONS

1 Historic cost – $75 000  12/15 = $60,000


Current cost – $80 000  12/15 = $64 000
2 Historical cost accounting makes extensive use of subjective estimates, for example, useful lives of
assets, allowances for doubtful receivables and inventory valuation.
It can be argued that some users are confused by historical cost accounts. Non-accountants may
believe that the statement of financial position shows the actual value of an entity's assets and
liabilities, rather than their cost.
3 The approach is to prepare a CPP statement of profit or loss.
$c $c
Sales (6000  360/330) 6 545
Less: cost of goods sold (5000  360/300) 6 000
545
Loss on holding cash for 6 months* (545)
Gain by owing payables for 1 month** 60
485
CPP profit 60

* ($6000  360/330) – $6000 = $c 545


**($2100  360/350) – $2100 = $c 60
Note that under historic cost accounting the gross profit would be $1000 ($6000 – $5000).
4 (a) The directors are probably justified in fearing that the fall in operating profit will have an
adverse effect on the share price in the short term, particularly as investors often look at
earnings per share in isolation.
However, the company is still profitable. A contingent liability for the same amount was
disclosed two years ago, so analysts will have had some warning that this situation might arise.
It should also be clear that this is a 'one off' expense and that otherwise the company has
performed strongly during the year. In the medium term the company's share price will
probably recover. There may be advantages in getting the bad news over quickly; analysts may
feel that it is better to be aware of the worst case scenario.
(b) Advantages of publishing an environmental and social report:
 It would probably enhance the company's reputation. Hemlock would appear to be
responding to the information needs of all its shareholders, not just providers of capital.
 Ethical and 'green' issues are becoming increasingly important to customers, suppliers,
employees and the general public. If Hemlock is genuinely able to demonstrate that it is
aware of its corporate social responsibilities this may improve its performance and share
price in the longer term.
Disadvantages of publishing an environmental and social report:
 If the report is to be taken seriously and to reflect well upon the company it must not be
perceived as a public relations exercise but a genuine attempt to provide useful information.
It must be at least as good as those of the company's competitors, otherwise it may damage
Hemlock's reputation, rather than enhance it.
 The report will be time consuming and costly to prepare. It will not be possible to 'pull
something together'. To be credible, the report should include performance indicators and
these should remain the same from year to year, even if performance declines in those areas.
FINANCIAL ACCOUNTING AND REPORTING | 147

5 The statement is incorrect. The directors' report does provide useful information for making
economic decisions, but that is not its only purpose. The information disclosed helps shareholders
to assess the stewardship of management. Investors need to know how efficiently and effectively
the directors have discharged their responsibility to use the entity's resources to generate returns.
In particular, the information about directors' remuneration is largely provided so that shareholders
will be aware of any attempts by the directors to reward themselves excessively.

MODULE 2
190 | FINANCIAL STATEMENTS

ANSWERS TO QUICK REVISION QUESTIONS

1 B I and II only. III and V are non-current assets. Income received in advance is termed deferred
income and recognised as a current (and/or non-current) liability.
2 B I and III only. II may be shown in the notes.
3 C Deferred tax liabilities and deferred tax assets, although the other items relate to the statement
of financial position, they do not need to be disclosed separately.
4 D The amount due within 12 months of the reporting date should be disclosed as a current
liability and the remainder as non-current.
5 D Non-controlling interests are apportioned their share of profits after tax; they do not feature
within the calculation of profit before tax.
6 B Both items have been dealt with correctly. The restructuring charge, although exceptional in
nature, should be charged against this year's profits. Although the adjustment to opening
inventory is a prior period adjustment (it corrects an error) it also affects the profit for the current
year because it affects cost of sales for both years. Cost of sales for the prior period is increased
and profit reduced; cost of sales for the current year is reduced and profit increased.
7 B Assets will increase as the receivable value is higher than the carrying amount of the inventory.
There is no effect on liabilities. Capital and reserves will increase as the inventory was sold for a
profit.
8 D A and B are transactions with shareholders and so reported in the statement of changes in
equity; C is realised income and so reported within profit or loss in the statement of profit or
loss and other comprehensive income.
9 C It includes other comprehensive income.
Other comprehensive income is included within the aggregated amount for total
comprehensive income. Changes in equity is a primary statement and includes gains taken
directly to reserves.
It is presented with the same prominence as the statement of profit or loss and statement of
financial position. All gains are included whether they are reported in the statement of profit or
loss or in reserves.
10 B $36m
$m
Profit for the year 29
Revaluations (14 – 7) 7
36

11 A Only the proceeds of a share issue and dividends received involve the movement of cash.
12 A Loss on sale of non-current assets should be added back to profit before tax.
13 C
$
Add: depreciation charge 980 000
Less: profit on sale of assets (40 000)
Less: increase in inventories (130 000)
Add: decrease in receivables 100 000
Add: increase in payables 80 000
Addition to operating profit 990 000

14 D Depreciation should be added back as it not a cash flow and proceeds from sale of non-current
assets appears under 'investing' cash flows.
FINANCIAL ACCOUNTING AND REPORTING | 191

15 D PROPERTY, PLANT AND EQUIPMENT


$'000 $'000
Opening balance 600 Disposals 90
Purchases (balancing figure) 240 Closing balance 750
840 840
Purchase of property, plant and equipment 240 000
Proceeds from sale of property, plant and equipment (30 000)
Net cash outflow 210 000

16 A
$'000
Issue of share capital (2500 – 1000) 1 500
Repayment of loans (1000 – 750) (250)
Net cash inflow 1 250

17 D Profit on disposal will be included in profit, so should be deducted.


18 C
$'000
Retained earnings 31.12.X5 1 660
Retained earnings 31.12.X4 1 470
 Post tax profit for 20X5 190
Add back tax charge* 840
1 030

* Note: This is the current liability at 31.12.X5, just as the 20X4 charge is the current liability at
31.12.X4.
19 A PROPERTY, PLANT AND EQUIPMENT: COST
$m $m
Opening balance 40 Transfer disposal (balancing figure) 6
Cash – additions 16 Closing balance 50

MODULE 3
56 56

PROPERTY, PLANT AND EQUIPMENT: ACCUMULATED DEPRECIATION


$m $m
Opening balance 10
Transfer disposal (balancing figure) 2 Profit or loss 6
Closing balance 14
16 16

PROPERTY, PLANT AND EQUIPMENT: DISPOSAL


$m $m
Transfer cost 6 Transfer depreciation 2
Loss on sale 1
Proceeds of sale (balancing figure) 3
6 6

20 A Interest paid = $38 000


INTEREST PAYABLE
$ $
Cash paid (bal fig) 38 000 Opening balance 12 000
Closing balance 15 000 Profit or loss 41 000
53 000 53 000
21 D
22 A It is the only one of the four options that is a gain or loss accounted for in 'other comprehensive
income' rather than in profit or loss for the year.
23 B Expressing an opinion on the financial statements is the responsibility of the external auditor.
192 | FINANCIAL STATEMENTS

ANSWERS TO MODULE QUESTIONS

1 SHABNUM CO.
STATEMENT OF CASH FLOWS FOR THE YEAR ENDED 31 DECEMBER 20X2
$'000 $'000
Cash flows from operating activities
Profit before tax 300
Depreciation charge (W1) 90
Interest expense 50
Loss on sale of property, plant and equipment (45 – 32) 13
Profit on sale of non-current asset investments (5)
(Increase)/decrease in inventories (48)
(Increase)/decrease in receivables (75)
Increase/(decrease) in payables 8
Cash generated from operating activities 333
Interest paid (75)
Tax paid (110 + 140 – 120) (130)
Net cash from operating activities 128
Cash flows from investing activities
Payments to acquire property, plant and equipment (W2) (201)
Payments to acquire intangible non-current assets (50)
Receipts from sales of property, plant and equipment 32
Receipts from sale of non-current asset investments 30
Interest received 25
Net cash used in investing activities (164)

Cash flows from financing activities


Proceeds from issue of share capital 60
Proceeds from long-term borrowings 120
Dividends paid (80)
Net cash from financing activities 100
Increase in cash and cash equivalents 64
Cash and cash equivalents at 1.1.X2 (note) (97)
Cash and cash equivalents at 31.12.X2 (note) (33)

NOTES TO THE STATEMENT OF CASH FLOWS


Note: Analysis of the balances of cash and cash equivalents as shown in the statement of financial
position.
20X2 20X1
$'000 $'000
Cash in hand 2 1
Short-term investments 50 –
Bank overdraft (85) (98)
(33) (97)
Workings
1 Depreciation charge
$'000 $'000
Accumulated depreciation at 31 December 20X2 340
Accumulated depreciation at 31 December 20X1 290
Depreciation on assets sold (85  45) (40)
(250)
Charge for the year 90
FINANCIAL ACCOUNTING AND REPORTING | 193

2 Purchase of property, plant and equipment


PROPERTY, PLANT AND EQUIPMENT
$'000 $'000
1.1.X2 Balance b/d 595 Disposals 85
Revaluation (OCI) 9
Purchases (bal fig) 201 31.12.X2 Balance c/d 720
805 805

Note: In this answer, dividends paid have been presented under financing activities, but it
would also be acceptable to include them under operating activities and/or investing activities.
2 The answer is C.
TRADE RECEIVABLES
$'000 $'000
At 1.1.X6 (balance) 50 Cash received 220
Irrecoverable debts 10
Sales 250 At 31.12.X6 70
300 300

3 The answer is $625 000.


TRADE PAYABLES AND ACCRUALS
$'000 $'000
At 31.12.X4 220 At 1.1.X4 130
Cash paid (balance) 625 Purchases and expenses (W) 715
845 845

Purchases:
$'000
Cost of sales: 575
Less: depreciation (40)

MODULE 3
535
Less: opening inventories (390)
Add: closing inventories 450
Purchases 595
Operating expenses 120
Purchases and expenses 715

4 The main disadvantages of cash flow accounting are essentially the advantages of accruals
accounting (proper matching of related items). There is also the practical problem that few
businesses keep historical cash flow information in the form needed to prepare a historical
statement of cash flows and so extra record-keeping is likely to be necessary.
A problem common to all financial statements is that cash flow information is historic, however
users are more interested in forward looking information. A further drawback is the possibility of
manipulation of cash flows. For example, a business may delay making large payments to suppliers
until after the end of the accounting period.
FINANCIAL ACCOUNTING AND REPORTING | 265

ANSWERS TO QUICK REVISION QUESTIONS 1

1 A Deferred development expenditure b/f is $480 000 (cost $500 000 – accumulated amortisation
$20 000), then deduct annual amortisation of $10 000 to give figure c/f of $470 000.
2 C I Development expenditure must be capitalised if the criteria are met.
II There is no time scale given by IAS 38 for amortisation.
3 D A and B are still in the research phase; C is in the development phase however problems mean
that it is still sufficiently distant from commercial production to write off expenses as incurred.
4 A Amortisation commences when production commences and therefore the amortisation charge
relates to an eight-month period. Amortisation should reflect the pattern of benefits; in this
case the first year's benefit is half that of later years, therefore:
0.5/9.5  $210 000  8/12m = $7368
* There are 10 years of amortisation, although the first year attracts only half the charge of the
subsequent nine years. Therefore the denominator is 9.5, being (1 year  ½ ) + (9 years  1)
5 C Amortisation does not start until commercial production commences.
6 B Intangible assets such as capitalised development costs that meet the capitalisation criteria of
IAS 38 must be capitalised and depreciated where they have a finite life. Any intangible assets
which are deemed to have an indefinite life are instead tested annually for impairment. Any
amortisation is charged to profit or loss.
7 C The asset must be identifiable. An asset which is capable of separate disposal is identifiable
however separability is not an essential feature of an intangible asset. For example, production
rights may not be capable of separate sale without a particular machine, however they still
qualify as an intangible asset.
8 C Only intangible assets which have an indefinite useful life or are not yet available for use must
be reviewed for impairment annually. All others undergo an impairment review if there are
indications of an impairment.
9 B $900 000/25  5/12 months = $15 000 amortisation to the end of the year.
Therefore carrying amount is $900 000 – $15 000 = $885 000.
Marketing costs must be expensed as incurred.
10 A Patents are purchased intangible assets and as such should be capitalised. The costs of staff
training and advertising, even where these are expected to result in future economic benefits,
cannot be capitalised according to IAS 38.
11 B A revaluation surplus is recognised as other comprehensive income in the period in which it
arises. An impairment loss suffered on a previously revalued asset may be recognised as other
comprehensive income to the extent that a revaluation surplus exists in respect of that asset. An
MODULE 4

impairment loss suffered on an asset held at depreciated cost must be recognised in profit or
loss.
12 D The impairment would have been written off to profit or loss as there is no credit in the
revaluation surplus for this asset. So only the increase in head office value appears in the
revaluation surplus.
13 C Most non-current assets only require testing for impairment if there are indicators of
impairment, rather than annually.
14 A The impairment loss is applied first against the goodwill and then against the other non-current
assets on a pro-rata basis. It will be allocated as follows:
$m
Building 10
Plant and equipment 5
Goodwill 5
20

The carrying amount of the building will then become $10m (20 – 10).
266 | APPLICATION OF SPECIFIC ACCOUNTING STANDARDS

15 C Goodwill acquired in a business combination should be reviewed for impairment annually.


16 B
Revaluation
Property reserve
$ $
Cost 1 April 20X5 700 000
Depreciation (700 000  2 / 50) (28 000)
CV at 1 April 20X7 672 000
Revaluation surplus 288 000 288 000
960 000
Depreciation y/e 31 March 20X8
(960 000 / 48) (20 000)
Excess depreciation (20 – 14) 000 000 (6 000)
CV at 31 March 20X8 940 000 282 000
Impairment loss (340 000) (282 000)
Impaired value 600 000

17 C
Revaluation
reserve
$ $
Cost 300 000
Revaluation 50 000 50 000
X3 350 000
Revaluation 50 000 50 000
X4 400 000 100 000
Impairment (115 000) (100 000)
FV less costs of disposal 285 000 –

The balance of the impairment loss ($15 000) is charged to profit or loss.
18 A A reversal of an impairment loss in relation to goodwill cannot be recognised.
19 C The carrying amount of the machine on the date of the impairment test is $68 000 ($80 000 
8.5 / 10). The recoverable amount is $66 000 (the higher of the value in use and the fair value
less costs of disposal). Therefore the asset is impaired and its carrying amount is reduced to
$66 000. The depreciation charge for the following year is therefore $66 000 / 8.5 years = $7,765
20 D An intangible asset with an indefinite useful life is tested for impairment annually; however the
test may take place at any time during the reporting period providing that it is the same time
each year.
A head office building which is shared by a number of CGUs (a corporate asset) is only tested
for impairment on an individual basis if it cannot be allocated to CGUs on a reasonable and
consistent basis.
FINANCIAL ACCOUNTING AND REPORTING | 267

ANSWERS TO QUICK REVISION QUESTIONS 2

1 B No sale has taken place, so DT must show that it is holding $90 000 which belongs to XX.
2 C The revenue on the system can be recognised immediately of $3560 (4000  89%) and the
revenue on the servicing of $440 (4000 x 11%) should be spread over the year.
$ %
System 4000 89
Servicing 500 11
3 B Settlement discounts count as variable consideration and any expected discounts should be
deducted from revenue.
Tuition fees are revenue from the provision of a service and should be recognised over the
period that the tuition is provided.
$
4 C Tax charge (30%  $582 000) 174 600
Over-provision from X7 ($502 000  30%) – 149 000 (1 600)
173 000

5 C The deferred tax liability will be the tax on temporary differences at the year end, i.e.
((2 650 000 – 1 872 000)  20%) = $155 600. Answer A is the deferred tax part of the year's tax
charge (the amount by which the liability is increased) i.e. $155 600 – $128 500. Answer B is the
opening balance rather than the year end balance. Answer D is the difference between the
carrying amount and the tax WDV, i.e. the temporary difference itself rather than the tax on the
difference.
6 D The creation of a deferred tax liability will increase the tax charge, so reducing profit after tax
for the year that is, the entry will be Dr. tax charge, Cr. deferred tax liability. The net assets will
also decrease due to increase in deferred tax liability.
7 A A carrying amount of $3 570 000 less tax WDV of $2 450 000 = $1 120 000 and at 22 per cent a
liability of $246 400 is required. There is already a liability of $250 000 so a credit of $3600 will be
reported.
8 A $
Current tax 53 960
Deferred tax (see below) (5 600)
48 360
Deferred tax – liability required = $80 000  24%
= 19 200
Opening balance on deferred tax = 24 800
MODULE 4

Decrease 5 600

9 B Deferred tax is recorded as a non-current liability; it does not represent an amount currently
payable, but is an accounting adjustment made in order to 'smooth' the tax charge so that the
tax charged in a period is in line with the profits made in that period.
10 A A deferred tax asset arises in respect of the losses carried forward which are expected to be
utilised against future profits that is, $185 000 ($320 000 – $120 000 – $15 000). The asset is
calculated by applying the tax rate of 25 per cent.
11 C A deferred tax liability is the result of a taxable temporary difference. Deferred tax arising on a
revaluation is recognised in other comprehensive income.
268 | APPLICATION OF SPECIFIC ACCOUNTING STANDARDS

12 A Deferred tax on the accelerated capital allowances (recognised in profit or loss)


$
Carrying amount (580 000 – 80 000) 500 000
Tax base (580 000 – 130 000) 450 000
Temp difference 50 000
Deferred tax liability c/f (20%  $50 000) 10 000
Deferred tax liability b/f 9 000
Increase in liability/charge to profits 1 000
Deferred tax on revaluation (recognised as OCI)
$100 000  20% 20 000

13 A $
Tax charge for current year = liability 56 000
Decrease in deferred tax liability (6 000)
Over-provision (balancing figure) (2 500)
Tax charge to profits 47 500
FINANCIAL ACCOUNTING AND REPORTING | 269

ANSWERS TO QUICK REVISION QUESTIONS 3

1 C The functional currency of an entity is the currency in which its day to day transactions are
denominated; the presentation currency is chosen by the management and is the currency in
which the financial statements are prepared.
2 A 600 000 crowns / 7 = $85 714
The closing rate must be used.
3 C $
Inventory value at 1 November 20X6 (450 000  $1.5) 675 000
Inventory value at 31 December 20X6 (450 000  $1.45) (652 500)
Gain / (Loss) 22 500
The inventory is a non-monetary asset. It is translated at the date of the original transaction and
remains in the statement of financial position at that amount being $300 000.
4 B Subsidiary B Co. is clearly an extension of Parent's Co.'s own activities and therefore it almost
certainly has the same functional currency as the parent.
5 D Exchange gain:
$
Purchase from European supplier:
1 June 20X4 (250 000  1.6) 400 000
30 June 20X4 (250 000  1.61) (402 500)
Gain / (Loss) (2 500)
Plus translation gain of 20 000
Total consolidated exchange gain 17 500

The overseas subsidiary has a different functional currency from its parent (i.e. it is a
semi-autonomous operation) and therefore the translation gain of $20 000 is recognised in
consolidated reserves rather than reported in profit or loss.
6 A The assets and liabilities of a foreign subsidiary are always translated at closing rate, not historic
rate, where the subsidiary has a different functional currency from its parent. For the translation
of a foreign subsidiary there is no consideration of monetary or non-monetary assets and
liabilities, all are translated at the closing rate.
7 B $ $
Opening net assets at opening rate (410 000 / 2) 205 000
Opening net assets at closing rate (410 000 / 1.8) 227 778
Gain 22 778
Profit for year at average rate (500 000 – 410 000) / 1.9 47 368
MODULE 4

Profit for year at closing rate (90 000 / 1.8) 50 000


Gain 2 632
Overall exchange gain 25 410

8 A $
Total assets (630 000 / 3) 210 000
Share capital (100 000 / 2) 50 000
Reserves (bal) 100 000
Current liabilities (180 000 / 3) 60 000
Equity and liabilities 210 000
270 | APPLICATION OF SPECIFIC ACCOUNTING STANDARDS

9 C Exchange difference on settled amount:


$
MP 130 000 / 2.3 56 522
MP 130 000 / 2.1 61 905
Loss 5 383
Exchange difference on retranslation of outstanding amount at year end:
$
MP 100 000 / 2.3 43 478
MP 100 000 / 1.8 55 556
Loss 12 078
Overall loss 17 461

10 C Unrealised exchange differences on the retranslation of monetary items in individual entity


financial statements are recognised in profit or loss, not other comprehensive income.
Monetary items do not include current assets such as inventories and prepayments, which are
classified as non-monetary items.
Non-monetary items denominated in a foreign currency have to be retranslated before they can
be included in the statement of financial position.
FINANCIAL ACCOUNTING AND REPORTING | 271

ANSWERS TO QUICK REVISION QUESTIONS 4


1 C
Dates Narrative $
1.1.20X4 Initial recognition 24 000
31.12.20X4 Interest 2 880
(24 000  12%)
31.12.20X4 Instalment (14 160)
Liability 12 720
2 A For leases that are less than 12 months or that have a low value at inception the business can
elect to use the simplified accounting under IFRS 16.
3 B $2 500 000  10% = $250 000
4 B 2 150 000 – 1 765 000 = $385 000
Dates Narrative $
1.1.20X4 Initial recognition 2 500 000
31.12.20X4 Interest 250 000
31.12.20X4 Instalment (600 000)
Liability 2 150 000
Interest (2 150 000  10%) 215 000
Instalment (600 000)
31.12.20X5 Liability 1 765 000
5 D The right of use asset is initially recognised at $75 000 (10 000 + 65 000) and this is depreciated
over six years to give a depreciation charge of $12 500 ($75 000 / 6), giving a carrying value of
$62 500 ($75 000 – $12 500). The asset is depreciated over six years rather than five years as the
lessee takes ownership at the end of the lease term.

MODULE 4
272 | APPLICATION OF SPECIFIC ACCOUNTING STANDARDS

ANSWERS TO MODULE QUESTIONS

1 Project A
This project meets the criteria in IAS 38 for development expenditure to be recognised as an asset.
The project is technically feasible and the company intends to complete it so that the resulting
product will be available for sale. There is a market for the product and its sale will result in future
economic benefits. In the absence of other information it is assumed that the company has
adequate technical, financial and other resources to complete the development and to use or sell
the intangible asset. The company is clearly able to measure reliably the expenditure attributable
to the intangible asset during its development, because it has been able to estimate total costs
and expected revenue.
Hence the costs of $280 000 incurred to date should be transferred from research and
development costs to capitalised development expenditure and carried forward until revenues are
generated; they should then be matched with those revenues.
Project B
While this project meets most of the criteria discussed above which would enable the costs to be
carried forward it fails on the requirements that 'adequate resources exist, or their availability can
be demonstrated, to complete the project'.
Therefore, these costs should be written off as an expense in profit or loss. Once funding is
obtained the situation can then be reassessed and future costs may be capitalised.
Project C
This is a research project according to IAS 38, i.e. original and planned investigation undertaken
with the prospect of gaining new scientific or technical knowledge or understanding.
There is no certainty as to its ultimate success or commercial viability and therefore it cannot be
considered to be a development project. IAS 38 therefore requires that costs be written off as
incurred.
2 (a) STATEMENT OF PROFIT OR LOSS AND OTHER COMPREHENSIVE INCOME (EXTRACT)
$
Research expenditure (Project C + 1 420 000) 1 530 000
Development costs (Project B) 150 000
Amortisation of capitalised development costs 240 000
(b) STATEMENT OF FINANCIAL POSITION (EXTRACT)
$
Non-current assets
Intangible assets
Deferred development costs 1 280 000
(c) NOTE TO FINANCIAL STATEMENTS
Deferred development costs
$
Cost
Balance b/f 1 480 000
Additions during year (Project A) 280 000
Balance c/f 1 760 000
Amortisation
Balance b/f 240 000
Charge during year 240 000
Balance c/f 480 000
Net carrying amount at 30 September 20X5 1 280 000
Net carrying amount at 30 September 20X4 1 240 000
FINANCIAL ACCOUNTING AND REPORTING | 273

3 Here are two possibilities:


(a) A mining company owns a private railway that it uses to transport output from one of its mines.
The railway now has no market value other than as scrap, and it is impossible to identify any
separate cash inflows with the use of the railway itself. Consequently, if the mining company
suspects an impairment in the value of the railway, it should treat the mine as a whole as a cash
generating unit, and measure the recoverable amount of the mine as a whole.
(b) A bus company has an arrangement with a town's authorities to run a bus service on four routes
in the town. Separately identifiable assets are allocated to each of the bus routes, and cash
inflows and outflows can be attributed to each individual route. Three routes are running at a
profit and one is running at a loss. The bus company suspects that there is an impairment of
assets on the loss-making route. However, the company will be unable to close the loss-making
route, because it is under an obligation to operate all four routes, as part of its contract with the
local authority. Consequently, the company should treat all four bus routes together as a cash
generating unit, and calculate the recoverable amount for the unit as a whole.
4 In identifying Minimart's cash-generating unit, an entity considers whether, for example:
 Internal management reporting is organised to measure performance on a store-by-store basis.
 The business is run on a store-by-store profit basis or on a region or city basis.
All Maximart's stores are in different neighbourhoods and probably have different customer bases.
So, although Minimart is managed at a corporate level, Minimart generates cash inflows that are
largely independent from those of Maximart's other stores. Therefore, it is likely that Minimart is a
cash-generating unit.
5 The reversal of the impairment loss is recognised to the extent that it increases the carrying
amount of the tangible non-current assets to what it would have been had the impairment not
taken place, i.e. a reversal of the impairment loss of $10m is recognised and the tangible non-
current assets written back to $70m. Reversal of the impairment is not recognised in relation to the
goodwill and patent because the effect of the external event that caused the original impairment
has not reversed – the original product is still overtaken by a more advanced model.
6 A. Dr. Trade receivables control account $5 400
Cr. Sales $5 400
As Freddie expects the customer to take up the settlement discount the sale is recorded at 90%
of the $6 000.
B. Dr. Cash $6 000
Cr. Trade receivables control account $5 400
Cr. Sales $600
As the customer pays on 16 May 20X7 they will not be entitled to the settlement discount so
Freddie will receive an additional $600 of sales revenue.
7 The cleaning company should recognise the revenue as they satisfy the performance obligations,
MODULE 4

this will mean they should recognise them over a period of time. The contract requires them to
clean over a two year period so the total amount should be spread over the two years. For the year
ended 31 December 20X1 they will recognise revenue of $7500 ($15 000 / 2 years) and they will
have deferred income (shown under current liabilities) of $2500 ($10 000 – $7500).
8 The airline satisfies the performance obligation in January when they provide the flight to the
customer so revenue should be recognised in January.
9 (a) $
Estimated tax charge for the 20X8 year ($120 000  30%) 36 000
Under provision relating to the previous year 5 000
Tax expense 41 000

(b) $
Estimated tax charge for the 20X8 year ($120 000  30%) 36 000
Over provision relating to the previous year (5 000)
Tax expense 31 000
274 | APPLICATION OF SPECIFIC ACCOUNTING STANDARDS

10 (a) The tax base of the machine is $7000. As $3000 has already been deducted, $7000 will be
deducted in the future.
(b) The tax base of the interest receivable is nil. It will be fully taxable in the future, so that zero is
deductible in the future.
(c) The tax base of the trade receivables is $10 000. Since it has already been taxed in full, it is not
taxable in the future (and so it is deductible).
(d) The tax base of the loan is $1m. Since economic benefits are not taxable, the tax base is the
same as the carrying amount.
11 (a) The tax base of the accrued expenses is nil. Since whole amount will be deducted in the future,
tax base is $1000 – $1000 = $0.
(b) The tax base of the interest received in advance is nil. This is carrying amount of $10 000 less
$10 000 (the amount is not taxable in the future, since it's already taxed).
(c) The tax base of the accrued expenses is $2000. This is the carrying amount of $2000 less $0.
Since it was already deducted for tax purposes, it will not be deducted in the future.
(d) The tax base of the accrued fines and penalties is $100. Since it cannot be deducted in the
future, this is $100 – $0 = $100.
(e) The tax base of the loan is $1m. Since no tax consequences, the tax base is equal to the
carrying value.
12 Jonquil Co. will recover the carrying amount of the equipment by using it to manufacture goods for
resale. Therefore, the entity's current tax computation is as follows:
Year
20X1 20X2 20X3 20X4 20X5
$ $ $ $ $
Taxable income* 10 000 10 000 10 000 10 000 10 000
Depreciation for tax purposes 12 500 12 500 12 500 12 500 0
Taxable profit (tax loss) (2 500) (2 500) (2 500) (2 500) 10 000

Current tax expense (income) at 40% (1 000) (1 000) (1 000) (1 000) 4 000

* That is, nil profit plus $50 000 / 5 depreciation add-back.


The entity recognises a current tax asset at the end of years 20X1 to 20X4 because it recovers the
benefit of the tax loss against the taxable profit of year 20X5.
The temporary differences associated with the equipment and the resulting deferred tax asset and
liability and deferred tax expense and income are as follows:
Year
20X1 20X2 20X3 20X4 20X5
$ $ $ $ $
Carrying amount 40 000 30 000 20 000 10 000 0
Tax base 37 500 25 000 12 500 0 0
Taxable temporary difference 2 500 5 000 7 500 10 000 0

Opening deferred tax liability 0 1 000 2 000 3 000 4 000


Deferred tax exp (income): bal fig. 1 000 1 000 1 000 1 000 (4 000)
Closing deferred tax liability  40% 1 000 2 000 3 000 4 000 0

The entity recognises the deferred tax liability in years 20X1 to 20X4 because the reversal of the
taxable temporary difference will create taxable income in subsequent years.
FINANCIAL ACCOUNTING AND REPORTING | 275

The entity's statement of profit or loss is as follows:


Year
20X1 20X2 20X3 20X4 20X5
$ $ $ $ $
Income 10 000 10 000 10 000 10 000 10 000
Depreciation (10 000) (10 000) (10 000) (10 000) (10 000)
Profit before tax 0 0 0 0 0
Current tax expense (income) (1 000) (1 000) (1 000) (1 000) 4 000
Deferred tax expense (income) 1 000 1 000 1 000 1 000 (4 000)
Total tax expense (income) 0 0 0 0 0
Profit before tax for the period 0 0 0 0 0

13
(a) $
Income tax on profits (liability in the statement of FP) 45 000
Deferred taxation 16 000
Under-provision of tax in previous year $(40 500 – 38 000) 2 500
Tax on profits for 20X3 (profit or loss) 63 500

(b) Tax payable on 20X3 profits (liability) 45 000

14 (a) Tax expense for the year


$
(i) Tax on trading profits (30% of $1 200 000) 360 000
Tax on capital gain 18 000
Deferred taxation 20 000
398 000
Under-provision of taxation in previous years $(84 000 – 80 000) 4 000
Tax expense on profit for the period 402 000

(ii) Note: The statement of profit or loss and other comprehensive income will show the
following:
$
Profit before tax (1 200 000 + 60 000) 1 260 000
Income tax expense (402 000)
Profit for the year 858 000
(b)
Deferred taxation
Balance brought forward 100 000
Transferred from profit or loss 20 000
MODULE 4

Deferred taxation in the statement of financial position 120 000

The tax liability is as follows:


Payable on 1 May 20X5 $
Tax on profits (30% of $1 200 000) 360 000
Tax on capital gain (30% of $60 000) 18 000
Due on 1 May 20X5 378 000
276 | APPLICATION OF SPECIFIC ACCOUNTING STANDARDS

Summary
Current liabilities
Tax payable on 1 May 20X5 378 000
Non-current liabilities
Deferred taxation 120 000
Note: It may be helpful to show the journal entries for these items.
$ $
Dr. Tax expense (profit or loss) 402 000
Cr. Tax payable *382 000
Deferred tax 20 000
* This account will show a debit balance of $4000 until the under-provision is recorded,
since payment has already been made: (360 000 + 18 000 + 4000).
The closing balance will therefore be $378 000.
15 The purchase will be recorded in the books of White Cliffs Co. using the rate of exchange ruling on
30 September.
Dr. Purchases $25 000
Cr. Trade payables $25 000
Being the $ cost of goods purchased for €40 000 (€40 000 / €1.60/$1)
On 30 November, White Cliffs must pay €20 000. This will cost €20 000 / €1.80/$1 = $11 111 and the
company has therefore made an exchange gain of $12 500 – $11 111 = $1389.
Dr. Trade payables $12 500
Cr. Exchange gains: (profit or loss) $1 389
Cr. Cash $11 111
On 31 December, the year end, the outstanding liability will be recalculated using the rate
applicable to that date: €20 000 / €1.90/$1 = $10 526. A further exchange gain of $1974 has been
made and will be recorded as follows:
Dr. Trade payables $1 974
Cr. Exchange gains: (profit or loss) $1 974
The total exchange gain of $3363 will be included in the operating profit for the year ending
31 December.
On 31 January, White Cliffs must pay the second instalment of €20 000. This will cost it $10 811
(€20 000 / €1.85/$1).
Dr. Trade payables $10 526
Exchange losses: (profit or loss) $285
Cr. Cash $10 811
FINANCIAL ACCOUNTING AND REPORTING | 277

16 SUMMARISED STATEMENT OF FINANCIAL POSITION AT 31 DECEMBER 20X9


$ $
Non-current assets (carrying amount) ($1: €1) 300
Current assets
Inventories ($1: €1) 200
Receivables ($1: €1) 100
300
600
Equity and liabilities
Equity
Ordinary shares (€100/1.8) 55

Pre-acquisition reserves (€120/1.8) 67


Post-acquisition reserves (balancing figure) 258
380
Non-current liabilities ($1: €1) 110
Current liabilities ($1: €1) 110
600

SUMMARISED STATEMENT OF PROFIT OR LOSS FOR THE YEAR ENDED 31 DECEMBER 20X9
$
Profit before tax (€300/1.6) 187.5
Tax (€140/1.6) (87.5)
Profit after tax, retained 100

17 STATEMENT OF PROFIT OR LOSS FOR THE YEAR ENDED 31 DECEMBER 20X7


$
Depreciation expense 4 845
Finance cost 2 187
STATEMENT OF FINANCIAL POSITION AS AT 31 DECEMBER 20X7
$
Non-current asset
Right of use asset (24 225 – 4845) 19 380
Non-current liability
Lease liability (20 412 – 6000) 14 412
Current liability
Lease liability 6 000
Working
The right of use asset will initially be recorded as follows:
Dr. Right of use asset $24 225
MODULE 4

Cr. Lease liability $18 225


Cr. Cash $6 000
The present value of the future cash flows ignores the first payment as this is paid as the lease is
taken out.
Lease liability calculation
Date Narrative $
1.1.20X7 Initial recording 18 225
31.12.20X7 Interest (18 225  12%) 2 187
20 412
Depreciation
24 225 / 5 years = $4845
346 | BUSINESS COMBINATIONS

ANSWERS TO QUICK REVISION QUESTIONS 1

400 000 shares


1 B Shareholding in Jamee = = 40%
1000 000 shares
With such a shareholding and with one director on the board of Jamee it is likely that Harvert
has significant influence over the operating and financial policies of Jamee (although not
control).
Therefore, this investment would be treated as an associate in the consolidated financial
statements of Harvert.
2 C A controls B and D as a result of holding the majority of the shares in each company, therefore
they are subsidiaries. B has significant influence over C and therefore C is an associate of B. C is
not controlled by A or B and is therefore not a subsidiary of either of these companies.
3 D The company is no longer controlled; the laws passed by the government transfer control to the
state from the majority shareholder.
4 C Representation of the parent on the board of directors of the investee is evidence of significant
influence rather than dominant influence or control.
5 B With a shareholding of 44 per cent it would not appear that Orius has control of Eerus but
particularly with representation on the board of directors it does seem to exert significant
influence. Therefore, Eerus would be treated as an associate using equity accounting.
6 C Provision of essential technical information is usually evidence of the existence of significant
influence by an entity. An associate is accounted for using the equity method. An associate
relationship is presumed where the holding of ordinary voting shares is at least 20 per cent, so
the answer is not A. A further indicator of an associate relationship is the provision of
management personnel – not simply operational personnel, hence not B. Where the parent has
the power to govern the financial policies of the investee by agreement as noted in D, this is
evidence of a parent-subsidiary relationship. Hence, consolidation is required.
7 B A subsidiary need not prepare accounts to the same date as the parent company in order for
them to be consolidated, however the subsidiary's reporting date should be within three
months either side of the group reporting date.
8 B II is untrue because consolidated accounts are only required where there is at least one
subsidiary, not where there is an associate only.
III is untrue, as equity accounting is only required where there is significant influence over the
investee, presumed to be the case when 20 per cent of the voting shares are held.
FINANCIAL ACCOUNTING AND REPORTING | 347

ANSWERS TO QUICK REVISION QUESTIONS 2

33.3
1 A (15 000 – 12 000)   80% = $599.54, rounded to $600
133.3
2 D $ $
Consideration transferred 180 000
Net assets acquired
Share capital 100 000
Retained earnings 90 000
190 000
Group share (190 000 × 80%) (152 000)
Goodwill attributable to parent 28 000
Goodwill attributable to non-controlling interest 4 000
Goodwill in statement of financial position 32 000

3 D Non-controlling interest = 20% × $260 000 = $52 000 + goodwill of $4 000 = $56 000
4 B $
Alpha retained earnings 210 000
Beta – group share post-acquisition (160 000 – 90 000) × 80% 56 000
266 000
5 C
$'000
Consolidated statement of financial position 445
Strachey (420)
25
Add: provision for unrealised profit (60  20/120) 10
35
6 A
$m
Falcon’s reserves 58
Kestrel’s reserves [80%  ($25m – $20m)] 4
Less: impairment of goodwill (8)
Consolidated reserves at 31 December 20X8 54

$'m
Goodwill workings:
Consideration transferred 24
Non-controlling interest at acquisition (20% x $20m) 4
28
Fair value of net assets acquired (20)
Goodwill at acquisition 8

7 C
$'000
Consideration transferred 400
Non-controlling interest at acquisition (10% x $350 000) 35
435

Net assets acquired [(300 000 shares @ $1 per share) + $50,000] (350)
Goodwill 85
MODULE 5

8 B $3100 to be included as cash in transit. The payment sent by STV the parent company to its
subsidiary TUW should be included as cash in transit on consolidation.
348 | BUSINESS COMBINATIONS

9 C
$'000 $'000
Net assets acquired:
Ordinary shares 300
Retained earnings at 1 January 20X1 80
Retained profit for the 9 months ended 30 September 20X1
(9/12  40) 30
Net assets at acquisition 410

Group share (80%) 328


Add: goodwill 20
348
10 B
$'000
XY 160
PQ 90
Inventory in transit 10
Provision for unrealised profit (20 + 10)  30% (9)
251
FINANCIAL ACCOUNTING AND REPORTING | 349

ANSWERS TO QUICK REVISION QUESTIONS 3

1 B $
Unrealised profit ($500 000 – $400 000) (100 000)
Add back: Additional depreciation (100 / 4) 25 000
Increase / (decrease) adjustment to consolidated profit or loss ( (75 000)

Pro forma consolidation worksheet entries:


Dr. Cr.
$ $
Unrealised profit 100 000
Non-current asset 100 000

Accumulated depreciation 25 000


Depreciation expense 25 000
75 000 75 000
2 C The amount of unrealised profit is $5000 (30 000  20/120).
The subsidiary has sold to the parent, therefore the unrealised profit has arisen in the accounts
of the subsidiary and must be allocated between the parent and the non-controlling interest.
3 D Revenue is reduced by the full amount of intra-group sales.
4 B Gross profit is reduced by the element of unrealised profit, which is 10 000  25 / 125.
5 C $'000
Profit before tax of subsidiary 55 000
Less: provision for unrealised profit (15 000  20/120  ½) (1 250)
53 750
NCI share (20%) 10 750

6 A 'Negative goodwill' (referred to as a bargain purchase in IFRS 3) is re-assessed and then


recognised immediately in profit or loss.
7 D $ $
Dr. Revenue 100 000
Cr. Cost of sales 100 000
Intra-group revenue must be eliminated in full from revenue as income and costs are wholly
intra-group. As there is no unsold inventory at the year end and, therefore, no unrealised profit,
the adjustment to costs is the same as the adjustment to revenue.

8 C $ $
Profit of Lay Co. 189 000
Profit of Hay Co. since acquisition 11 / 12  $60 000 55 000
Goodwill impairment
Consideration transferred 450 000
NCI (10%  460 000) 46 000
Net assets of Hay Co. (460 000)
(18 000)
Profit for the year 50% × 36 000 226 000

9 C $
NCI in Stereo's profit ($40 000  15%) 6 000
NCI share of impairment loss ($6 000  15%) (900)
MODULE 5

5 100

The unrealised profit is dealt with in the selling company's books. In this case that is Radio.
350 | BUSINESS COMBINATIONS

ANSWERS TO QUICK REVISION QUESTIONS 4

1 B All of a subsidiary's assets and liabilities are included in the consolidated statement of financial
position and then an amount to reflect those not owned is shown in the form of the non-
controlling interest.
Share capital in the consolidated statement of financial position is only ever the investor's share
capital.
2 B Consul cannot exercise significant influence over Warrior because it is controlled by another
entity which ignores Consul's views. However, it has the largest shareholding and a board seat,
so exercising significant influence over Admiral. Sultan is not so clear. As another entity also has
significant influence over Sultan, it is likely that Consul does too.
3 D
$m
Cost of investment 12.0
Group share of post-acquisition reserves (30%  5) 1.5
13.5
4 A Group gross profit will only include the gross profit of the parent and the subsidiary. The
group's share of the profit of the associate is reported on a separate line below gross profit in
the consolidated statement of profit or loss.
$'000
Savoy 700
Spring 550
1 250
5 C Statement (I) True – if K controls L then L will be a subsidiary, not an associate.
Statement (II) Untrue – there is a presumption that L would be an associate of K but this is
rebuttable for example, if another party held say 70 per cent of the shares
while K only held 30 per cent.
Statement (III) True – there is no elimination of balances for an associate as the associate is
not part of the group.
6 B Mark up on cost = 20 000 / 80 000  100%
= 25%
Unrealised profit = $16 000  25 / 125  25%
= $800
7 A
$
Investment brought forward 6 600 000
Profit after tax (420 000 – 180 000) 240 000
Dividends paid (bal fig) (90 000)
Investment carried forward 6 750 000

8 C Share of profit of associate: 30%  8 / 12  $66 000 = $13 200


Carrying amount of associate: $400 000 + $13 200 = $413 200
9 A
$
Income from associate (20%  $180 000) 36 000
Impairment in associate (see below) (26 300)
9 700

Cost of investment 420 000


Share of post acquisition profits (20%  530 000) 106 000
526 000
Impairment (5%  526 000) 26 300
FINANCIAL ACCOUNTING AND REPORTING | 351

10 B
$
AB's inventory 90 000
CD's inventory 38 000
128 000

Remember that the associate's inventory is not added across on a line by line basis; therefore
EF's $65 000 is not included in group inventory.
The unrealised profit to be eliminated is 30% × 25% × $20 000 = $1500, so a consolidation
adjustment to debit the share of profit of associates and credit the investment in associates by
$1500 is required. However this has no input on group inventory figures.

MODULE 5
352 | BUSINESS COMBINATIONS

ANSWERS TO MODULE QUESTIONS

1
INVESTMENT CRITERIA REQUIRED TREATMENT IN
GROUP ACCOUNTS
Subsidiary Control (> 50 per cent rule) Full consolidation (IFRS 10)
Associate Significant influence (20 per cent + rule) Equity accounting (IAS 28)
Investment which is neither of the No significant influence (< 20 per cent As for single company
above rule) accounts

2 The group structure is:


P Co.
60 per cent
S Co.
Partly eliminating items are the components of P Co.'s investment in S Co., that is, ordinary shares
and loan stock. Non-controlling shareholders have an interest in 40 per cent (8000/20 000) of
S Co.'s ordinary shares, including reserves.
You should now total the assets and liabilities and produce workings for non-controlling interest,
revaluation surplus and retained earnings as follows:
Workings
i Revaluation surplus
$
P Co. –
Share of S Co.'s revaluation surplus (60%  6 000) 3 600
3 600

ii Retained earnings
$
P Co. 22 000
Share of S Co.'s retained earnings (60%  4 000) 2 400
24 400

iii Non-controlling interest


$
S Co.'s net assets (66 000 – 36 000) 30 000
 40% 12 000
The results of the workings are now used to construct the consolidated statement of financial
position.
P GROUP
CONSOLIDATED STATEMENT OF FINANCIAL POSITION
$ $
Assets
Property, plant and equipment 65 000
Current assets 53 000
Total assets 118 000
Equity and liabilities
Equity attributable to parent
Ordinary shares 40 000
Revaluation surplus (W1) 3 600
Retained earnings (W2) 24 400
68 000
Non-controlling interest (W3) 12 000
80 000
FINANCIAL ACCOUNTING AND REPORTING | 353

$ $
Non-current liabilities
10% loan stock (26 000 – 8 000) 18 000
Current liabilities 20 000
Total equity and liabilities 118 000

Notes
a. S Co. is a subsidiary of P Co. because P Co. owns 60 per cent of its ordinary capital.
b. As always, the share capital in the consolidated statement of financial position is that of the
parent alone. The share capital in S Co.'s statement of financial position was partly eliminated
against the investment shown in P Co.'s statement of financial position, while the uneliminated
portion was credited to the non-controlling interest.
c. The figure for the non-controlling interest comprises the interest of outside investors in the
share capital and reserves of the subsidiary. The remaining portion of S Co.'s loan stock is not
shown as part of the non-controlling interest but is disclosed separately as a liability of the
group.
3 A The non-controlling interest at any given reporting date is measured as:
$
NCI as measured at acquisition X
NCI share of profits made by the subsidiary since acquisition X
X
Therefore:
NCI as measured at acquisition 460 000
NCI share of post-acquisition profits (10%  (275 400 + 286 000)) 56 140
516 140

4 P CO.
CONSOLIDATED STATEMENT OF FINANCIAL POSITION AS AT 30 JUNE
$ $
Assets
Non-current assets
Property, plant and equipment (120 000 + 100 000) 220 000
Current assets
Inventories (50 000 + 60 000) 110 000
Goods in transit (18 000 – 12 000) 6 000
Receivables (40 000 + 30 000) 70 000
Cash (4 000 + 6 000) 10 000
196 000
Total assets 416 000
Equity and liabilities
Equity
Ordinary shares, fully paid (parent) 100 000
Retained earnings (95 000 + 28 000) 123 000
223 000
Non-current liabilities
10% loan stock 75 000
12% loan stock (50 000  60%) 30 000
105 000
Current liabilities
Payables (47 000 + 16 000) 63 000
Taxation (15 000 + 10 000) 25 000
88 000
Total equity and liabilities 416 000
MODULE 5

Note especially how:


a. The remaining loan stock in S Co. becomes a liability of the group
b. The goods in transit is the difference between the current accounts ($18 000 – $12 000)
c. The investment in S Co.'s shares is eliminated against S Co.'s share capital.
354 | BUSINESS COMBINATIONS

5 To prepare the consolidated statement of financial position, follow the steps below:
1. Agree current accounts
Ping Co. has goods in transit of $2000 making its total inventory $3000 + $2000 = $5000 and its
liability to Pong Co. $8000 + $2000 = $10 000.
Eliminate common items: these are the current accounts between the two companies of
$10 000 each.
2. Calculate goodwill
Goodwill
Group NCI
$ $
Consideration transferred (W3)/fair value of NCI 38 734 9 000
Net assets acquired as represented by:
Ordinary share capital 25 000
Revaluation surplus on acquisition 5 000
Retained earnings on acquisition 6 000
Intangible asset – brand name 5 000
41 000
Group/NCI % (32 800) (8 200)
Goodwill 5 934 800

This goodwill (total $6734) must be capitalised in the consolidated statement of financial
position.
3. Consideration transferred
$
Cash paid 30 000
Contingent consideration: 10 000 × 1/(1.072)* 8 734
38 734

* Note that the contingent consideration has been discounted at 7 per cent for 2 years (1 July
20X7 to 1 July 20X9).
However, at the date of the current financial statements, 30 June 20X8, the discount for one
year has unwound. The amount of the discount unwound is:
$
(10 000 × 1 / 1.07) – 8734 612
So this amount will be charged to finance costs in the consolidated financial statements and the
contingent consideration under liabilities will be shown as $9346 (8734 + 612).
4. Calculate consolidated reserves
Consolidated revaluation surplus
$
Ping Co | 12 000
Share of Pong Co.'s post acquisition revaluation surplus –
12 000
Consolidated retained earnings
Ping Pong
$ $
Retained earnings per question 26 000 28 000
Less: pre-acquisition (6 000)
Discount unwound – finance costs (612) 22 000
Share of Pong: 80% × $22 000 17 600
42 988
FINANCIAL ACCOUNTING AND REPORTING | 355

5. Calculate non-controlling interest at year end


$
Pong Co.'s net assets per question (65 000 – 7000) 58 000
Intangible asset (brand name) 5 000
63 000

NCI share 20% 12 600


Goodwill (W2) 800
13 400

6. Prepare the consolidated statement of financial position


PING CO.
CONSOLIDATED STATEMENT OF FINANCIAL POSITION AS AT 30 JUNE 20X8
$ $
Assets
Non-current assets
Property, plant and equipment ($50 000 + $40 000) 90 000
Intangible assets: goodwill (W2) 6 734
Brand name 5 000
Current assets
Inventories ($5000 + $8000) 13 000
Receivables ($16 000 + $7000) 23 000
Cash 2 000
38 000
Total assets 139 734
Equity and liabilities
Equity
Ordinary shares 45 000
Revaluation surplus (W4) 12 000
Retained earnings (W4) 42 988
99 988
Non-controlling interest (W5) 13 400
113 388
Current liabilities
Trade payables ($10 000 + $7000) 17 000
Contingent consideration (W3) 9 346
Total equity and liabilities 139 734

6 To prepare the consolidated statement of financial position, follow the steps below:
1 Goodwill
$ $
Consideration transferred 46 000
Net assets acquired as represented by
Share capital 30 000
Retained earnings 10 000
(40 000)
Goodwill 6 000

2 Retained earnings
P Co. S Co.
$ $
Retained earnings per question 45 000 22 000
Unrealised profit: 20%  $15 000 (3 000)
Pre-acquisition (10 000)
9 000
Share of S Co. 9 000
MODULE 5

Goodwill impairment loss (1 500)


52 500
356 | BUSINESS COMBINATIONS

P CO.
CONSOLIDATED STATEMENT OF FINANCIAL POSITION
$ $
Assets
Non-current assets
Property, plant and equipment 120 000
Goodwill (6000 – 1500) 4 500
124 500
Current assets (W1) 55 000
Total assets 179 500
Equity and liabilities
Equity
Ordinary shares 100 000
Retained earnings 52 500
152 500
Current liabilities (W2) 27 000
Total equity and liabilities 179 500

Workings
1 Current assets
$ $
In P Co.'s statement of financial position 40 000
In S Co.'s statement of financial position 30 000
Less: S Co.'s current account with P Co. eliminated (12 000)
18 000
58 000
Less: unrealised profit excluded from inventory valuation (3 000)
55 000
2 Current liabilities
$
In P Co.'s statement of financial position 21 000
Less: P Co.'s current account with S Co. eliminated (12 000)
9 000
In S Co.'s statement of financial position 18 000
27 000

7 Singe Co. has made a profit of $24 000 ($43 000 – $19 000) for the year. In the absence of any
direction to the contrary, this should be assumed to have arisen evenly over the year; $6000 in the
three months to 31 March and $18 000 in the nine months after acquisition. The company's
pre-acquisition retained earnings are therefore as follows:
$
Balance at 31 December 20X4 19 000
Profit for three months to 31 March 20X5 6 000
Pre-acquisition retained earnings 25 000

The consolidation workings can now be drawn up.


1 Goodwill
$ $
Consideration transferred 50 000
Net assets acquired
represented by
Ordinary share capital 10 000
Retained earnings (pre-acquisition) 25 000
35 000
Group share 80% (28 000)
Goodwill attributable to group 22 000
Goodwill attributable to NCI 3 000
25 000
FINANCIAL ACCOUNTING AND REPORTING | 357

2 Retained earnings
Hinge Co. Singe Co.
$ $
Per question 47 000 43 000
Pre-acquisition (see above) (25 000)
18 000
Share of Singe: $18 000  80% 14 400
61 400
3 Non-controlling interest at reporting date
$
Singe Co. net assets (73 000 – 20 000) 53 000
 20% 10 600
Goodwill 3 000
13 600

HINGE CO.
CONSOLIDATED STATEMENT OF FINANCIAL POSITION AS AT 31 DECEMBER 20X5
$ $
Assets
Property, plant and equipment 62 000
Goodwill (W1) 25 000
Current assets 128 000
Total assets 215 000
Equity and liabilities
Equity
Ordinary shares 100 000
Retained earnings (W2) 61 400
161 400
Non-controlling interest (W3) 13 600
175 000
Current liabilities 40 000
Total equity and liabilities 215 000

8 S Co.: carrying amount at disposal (at historical cost) = $4000  1½ / 4 = $1500


 Profit on disposal = $1100 (depreciation charge for the year = $500)
P Co.: carrying amount at disposal (at fair value) = $3000  1½ / 2 = $2250
 Profit on disposal for consolidation = $350 (depreciation for the year = $750)
The non-controlling interest would be credited with 20 per cent of both the profit on disposal and
the depreciation charge as part of the one line entry in the consolidated statement of profit or loss
and other comprehensive income.
9 Goodwill on consolidation of Kono Co.
$m $m
Consideration ($2.00  6m) 12.0
Fair value of net assets acquired
Share capital 8.0
Pre-acquisition reserves 4.4
Fair value adjustments
Property, plant and equipment (16.6 – 16.0) 0.6
Inventories (4.2 – 4.0) 0.2
13.2
Group share 75% (9.9)
Goodwill 2.1

Notes on treatment:
MODULE 5

1. Share capital and pre-acquisition profits represent the book value (carrying amount) of the net
assets of Kono Co. at the date of acquisition. Adjustments are then required to this book value
in order to give the fair value of the net assets at the date of acquisition. For short-term
monetary items, fair value is their carrying amount on acquisition.
358 | BUSINESS COMBINATIONS

2. IFRS 3 states that the fair value of property, plant and equipment should be determined by
market value.
3. Raw materials should be valued at their replacement cost of $4.2m.
4. The rationalisation costs cannot be reported in pre-acquisition results under IFRS 3 as they are
not a liability of Kono Co. at the acquisition date.
10 The unrealised profit on disposal which must be eliminated from the consolidated profit or loss:
$
Proceeds 26 000
Carrying amount at date of disposal 6/10  $40 000 (24 000)
2 000
The increase in depreciation charge which must be eliminated from the consolidated profit or loss:
8 months depreciation on historic cost to the group 8/12  $40 000 / 10 2 667
8 months depreciation on transfer price 8/12  $26 000 / 6 2 889
222
Therefore, the overall adjustment to the consolidated profit or loss is an decrease of $1778 ($2000 –
$222) to the consolidated profit.
11 The shares in S Co. were acquired three months into the year. Only the post-acquisition proportion
(9/12) of S Co.'s statement of profit or loss is included in the consolidated statement of profit or
loss.
P CO. CONSOLIDATED STATEMENT OF PROFIT OR LOSS FOR THE YEAR ENDED
31 DECEMBER 20X5
$
Revenue (170 + 60) 230 000
Cost of sales (65 + 27) (92 000)
Gross profit 138 000
Administrative expenses (43 + 9) (52 000)
Profit before tax 86 000
Income tax expense (23 + 6) (29 000)
Profit for the year 57 000

Profit attributable to:


Parent 49 800
Non-controlling interest (18  40%) 7 200
57 000
STATEMENT OF CHANGES IN EQUITY
Retained Non-controlling
earnings interest
$ $
Balance at 1 January 20X5 81 000 –*
Dividends paid (6000  40%) (12 000) (2 400)
Total comprehensive income for the year 49 800 7 200
Added on acquisition of subsidiary (W) – 58 400
Balance at 31 December 20X5 118 800 63 200

* All of S Co.'s profits brought forward are pre-acquisition.


Working
$
Added on acquisition of subsidiary:
Share capital 100 000
Retained earnings brought forward 40 000
Profits Jan-Mar 20X5 (24 000 – 18 000) 6 000
146 000
Non-controlling share 40% 58 400
FINANCIAL ACCOUNTING AND REPORTING | 359

12 BRODICK GROUP
CONSOLIDATED STATEMENT OF PROFIT OR LOSS FOR THE YEAR TO 30 APRIL 20X7
$'000
Revenue (1100 + 500) 1600
Cost of sales (630 + 300) (930)
Gross profit 670
Administrative expenses (105 + 150) (255)
Profit before tax 415
Income tax expense (65 + 10) (75)
Profit for the year 340

Profit attributable to:

Parent 332
Non-controlling interest (W1) 8
340
CONSOLIDATED STATEMENT OF CHANGES IN EQUITY (extracts)
Retained Non-controlling
earnings interest
$'000 $'000
Balance brought forward (W2, W3) 500 221.2
Dividends paid (30 000 – 24 000) (200) (6)
Total comprehensive income for the year 332 8
Balance carried forward 632 223.2

Workings
1 Non-controlling interests
$'000
In Lamlash (20%  40) 8

2 Retained earnings brought forward


Brodick Co. Lamlash Co.
$'000 $'000
Per question 460 106
Less: pre-aqn (56)
50
Share of Lamlash: 80%  50 40
500
3 Non-controlling interest brought forward
$'000
Share capital 1 000
Retained earnings 106
1 106
Non-controlling share 20% 221.2

Note: The carried forward figures can be proved as follows:


Retained earnings 584 + 80% (116 – 56) = 632
Non-controlling interest 20%  (1 000 + 116) = 223.2
13 PARENT CO.
CONSOLIDATED STATEMENT OF PROFIT OR LOSS
$'000
Profit before tax of Parent and Subsidiaries 95
Share of profits of associate (50 × 40%) 20
Profit before tax 115
MODULE 5

Income tax expense (35)


Profit attributable to the members of Parent Co. 80
360 | BUSINESS COMBINATIONS

PARENT CO.
CONSOLIDATED STATEMENT OF FINANCIAL POSITION
$'000
Assets
Tangible non-current assets 220
Investment in associate (note) 104
Current assets 100
Total assets 424
Equity and liabilities
Share capital 250
Retained earnings (W) 174
Total equity and liabilities 424

Note:
$'000
Investment in associate
Cost of investment 60
Share of post-acquisition retained earnings (W) 24
Loan to associate 20
104
Working
Parent and
Retained earnings Subsidiaries Associate
$'000 $'000
Per question 150 100
Pre-acquisition 40
Post-acquisition 60

Group share in associate


($60  40%) 24
Group retained earnings 174

14 CONSOLIDATED STATEMENT OF PROFIT OR LOSS


$
Share of profit of associate ((82 000 – 32 000) × 25%) 12 500

CONSOLIDATED STATEMENT OF FINANCIAL POSITION


$
Investment in associate 45 500

Working
$
Cost of investment 38 000
Share of post-acquisition retained earnings ((82 000 – 32 000 – 20 000)  25%) 7 500
45 500

15 J GROUP CONSOLIDATED STATEMENT OF FINANCIAL POSITION AS AT 31 DECEMBER 20X5


$'000
Non-current assets
Freehold property (W2) 3 570.0
Plant and machinery (795 + 375) 1 170.0
Investment in associate (W9) 475.2
5 215.2
Current assets
Inventory (W3) 855.0
Receivables (W4) 620.0
Cash (50 + 120) 170.0
1 645.0
Total assets 6 860.2
FINANCIAL ACCOUNTING AND REPORTING | 361

$'000
Equity and liabilities
Equity
Share capital 2 000.0
Retained earnings (W10) 1 776.2
3 776.2
Non-controlling interest (W11) 894.0
4 670.2
Non-current liabilities
12% loan stock (500 + 100) 600.0
Current liabilities (W5) 1 590.0
Total equity and liabilities 6 860.2

Workings
1 Group structure

2 Freehold property
$'000
J Co. 1 950
P Co. 1 250
Fair value adjustment 400
Additional depreciation (400  50% / 40)  6 years (20X0-20X5) (30)
3 570
3 Inventory
$'000
J Co. 575
P Co. 300
Provision for unrealised profit (100  25/125) (20)
855
4 Receivables
$'000
J Co. 330
P Co. 290
620
5 Current liabilities
$'000
J Co.: bank overdraft 560
trade payables 680
P Co.: trade payables 350
1 590
6 Provision for Unrealised Profit (PURP)
$'000
On sales to J (Parent Co.) 100  25/125 20.0

On sales to S (associate) 80  25/125  30% 4.8


7 Fair value adjustments
Difference at acquisition Difference now
$'000 $'000
Property 400 400
Additional depreciation: 200  6/40 – (30)
MODULE 5

400 370
 Charge $30 000 to retained earnings
362 | BUSINESS COMBINATIONS

8 Goodwill
$'000 $'000
P Co.
Consideration transferred 1 000
Net assets acquired
Share capital 1 000
Retained earnings 200
Fair value adjustment 400
1 600
Group share 60% (960)
Goodwill at acquisition 40
Impairment loss (40)
0
9 Investment in associate
$'000
Cost of investment 500.00
Share of post-acquisition profit (390 – 150)  30% 72.00
Less: PUP (W6) (4.80)
impairment loss (92.00)
475.20
10 Retained earnings
J P S
$'000 $'000 $'000
Retained earnings per question 1 460.0 885.0 390.0
Adjustments
Unrealised profit (W6) (20.0)
Fair value adjustments (W7) (30.0)
835.0 390.0
Less: pre-acquisition reserves (200.0) (150.0)
1 460.0 635.0 240.0
P: 60%  635 381.0
S: 30%  240 72.0
Less: PUP on sales to associate (W6) (4.8)
impairment losses: P (40.0)
S (92.0)
1 776.2
11 Non-controlling interest at reporting date
$'000
Net assets of P Co. 1 885.0
Fair value adjustment (W7) 370.0
Less PUP: sales to J Co. (20.0)
2 235.0
Non-controlling interest (40%) 894.0
FINANCIAL ACCOUNTING AND REPORTING | 397

ANSWERS TO QUICK REVISION QUESTIONS

1 A An increase in sales will probably lead to an increase in earnings before interest and tax
(sometimes called operating profit). There will be no increase in investment (net assets or

MODULE 6
capital employed).
Issuing ordinary shares (option B) increases net assets and decreases ROI in the short term,
although the issue proceeds can be used to generate additional profit and this may help to
increase ROI in the longer term. Revaluing land and buildings upwards (option C) decreases
ROI, because it increases net assets and reduces profits. Increasing the level of dividends
(option D) has no effect on ROI.
70 000
2 B Quick ratio: = 0.44: 1
159 000
$'000
Assets as stated (80 + 10) 90
Less customers taking advantage of discount (20)
70
Liabilities as stated (75 + 100) 175
Cash received from sale of inventory (8 – 2) (6)
Cash received from trade receivables (20  90%) (18)
Current portion of new loan (40 / 5) 8
159
3 D Example: suppose the entity purchases inventory worth $300 000:
Current ratio Quick ratio
Before 1 500 400
=1.5 = 0.4
1 000 1 000
After 1800 400
=1.4 = 0.3
1300 1300

Profit available to equity shareholders


4 B
Shareholders' equity
20 + 7.5
= 6.1%
300 + 150
5 C ABC has a relatively high operating profit and a relatively low asset turnover. This suggests that
it is capital intensive. A and D would not normally have high levels of non-current assets. B is
more likely to have significant non-current assets, but this type of business normally has low
operating profit share margins. Therefore, option C is the most likely option.
Interest bearing debt 100 + 50
6 C = = 45.5%
Shareholders'equity + interest bearing debt 330
7 C D and A are both likely to result in increased sales revenue relative to investment (capital
employed) and therefore in increased asset turnover. B is irrelevant.
Trade receivables 690
8 C Receivables collection period =  365 = 45 days
Sales 5600
Trade payables 250
Payables payment period =  365 = 20 days
Cost of sales 4500
If the cash cycle is 105 days, inventories turnover must be 80 days (105 + 20 – 45).
Inventories 80
Inventories turnover is so inventories are  4500 = $986 000
Cost of sales 365
398 | ANALYSIS OF FINANCIAL STATEMENTS

9 D MNO has no inventories, moderate levels of land and property, low levels of other non-current
assets and very high trade receivables and trade payables. This suggests that MNO operates in
a service industry. An insurance broker is the only one of the four that fits this profile.
10 D The P/E ratio provides indication of market confidence in an entity, not risk therefore both A
and C are incorrect. The ratio is calculated as price per share/earnings per share and therefore
without the share price of each entity it is impossible to decide whether B is correct. D is correct
as a low P/E may be due to a low share price.
11 B 1.26:1
(Receivables + short-term investments) are divided by (trade payables, overdraft, taxes payable
+ deposits in advance):(158,000 + 18,000)/(61,000 + 64,000 + 10,000 + 5,000) = 1.26
12 C Reducing the payables payment period will increase the length of a company's cash cycle, as
cash is being paid out more quickly. The other options will all speed up the cash cycle.
13 D 4.1%
(Dividends (3.4c + 11.1c) / Share price) × 100% = (14.5c / 350c) × 100% = 4.1%
14 B The new product will have an operating profit margin of 120 / 1,600 = 7.5%, so will reduce the
current margin of 10%. It will have an ROI of 120 / 500 = 24%, higher than the current 20%.
15 A The effect of this impairment will increase the ROI ratio of TH Co, and increase its gearing ratio.
Capital employed (assets) would decrease, increasing ROI. The impairment loss will reduce
equity(revaluation surplus) and so increase gearing.
16 C Obsolete inventory lines. Obsolete goods can lead to a build-up of unsold inventory, thereby
increasing the holding period. A reduction in selling price or an increase in demand could
increase sales leading to a fall in the holding period. Seasonal fluctuations will change the
holding period throughout the year, but should not affect the year on year picture.
17 A Renegotiating a loan to secure a lower interest rate will have no effect on gearing as the debt
amount will still be the same.
18 C Where loan notes are issued to replace shares and vice versa will have no impact on operating
profits or total capital employed and will therefore have no impact.
Revaluations decrease profits and increase capital employed and will therefore decrease return
on capital employed.
A year end investment in plant and machinery financed through additional capital will increase
the capital employed without increasing profits in the current year, and will therefore reduce
ROI.
ROI can be expressed as: asset turnover x operating margin. Therefore, when asset turnover
falls, this causes ROI to decrease.
19 A $1.5m
Revenue: current assets = 5:1
Therefore current assets = $30m/5 = $6m
Current ratio (current assets: current liabilities) = 2:1
Therefore current liabilities = $6m/2 = $3m
Acid test ratio (current assets – inventory: current liabilities) = 1.5:1
Therefore current assets – inventory = $3m × 1.5 = $4.5m
Hence, inventory = $6m – $4.5m = $1.5m
20 C P/E ratio = current market price per share/EPS so an increase in share price will cause the P/E
ratio to increase
Dividend yield = dividend per share/market price per share  100 so an increase in share price
will cause the dividend to fall.
21 B Receivable days – supermarkets do not offer customers a credit period. Immediate payment is
required on checkout. Therefore, there will be few trade receivables making receivable days a
pointless ratio to calculate.
FINANCIAL ACCOUNTING AND REPORTING | 399

22 B RT Co. moved to an out-of-town office location where rent and employment costs were lower
than they were in 20X8.
Rent and employment are both operating costs, so reducing them would improve the operating
margin.
Interest is a finance cost, and would not be included within operating profit.

MODULE 6
Increasing the trade discounts offered would actually reduce the operating margin because it
would increase the cost of discounts allowed.
Increased public awareness of the company might help to explain e.g. a higher level of sales
activity, but this would not have any impact on the operating profit margin.
23 C Non-current asset turnover
A manufacturing company will have high assets (factory, plant and machinery, inventories etc.).
Therefore non-current asset turnover (sales/non-current assets) will be a measure of how
efficiently an entity is using its non-current assets to generate revenue.
24 D P/E ratio
The P/E ratio is a measure of the market confidence in the future of an entity. Gearing relates to
long-term solvency and the current ratio relates to liquidity.
25 B There are no prior year figures to compare to current year figures is incorrect because in
published financial statements comparatives must be shown.
400 | ANALYSIS OF FINANCIAL STATEMENTS

ANSWERS TO MODULE QUESTIONS

1 20X8 20X7
EBIT 360 245 247 011
Interest payable 18 115 21 909
= 20 times = 11 times
Furlong has more than sufficient interest cover. In view of the company's low gearing, this is not
too surprising and so we finally obtain a picture of Furlong as a company that does not seem to
have a debt problem.
2
20X7 20X6
Current ratio 626.8 654.4
= 1.05 = 1.02
599.1 642.2
Quick ratio 584.1 576.4
= 0.97 = 0.90
599.1 642.2
Accounts receivable 295.2 335.5
collection period  365 = 50 days  365 = 52 days
2 176.2 2 344.8
Inventory turnover period 42.7 78.0
 365 = 9 days  365 = 16 days
1 659.0 1 731.5
Accounts payable 190.8 188.1
payment period  365 = 43 days  365 = 40 days
1 623.7 1 726

Since cost of sales = opening inventories plus purchases less closing inventories, purchases = cost
of sales less opening inventories plus closing inventories. For 20X7 purchases = 1659 – 78 + 42.7 =
1623.7. For 20X6 purchases = 1731.5 – 83.5 + 78 = 1726.
The company's current ratio is a little lower than average but its quick ratio is better than average
and slightly less than the current ratio. This suggests that inventory levels are strictly controlled,
which is reinforced by the low inventory turnover period. It would seem that working capital is
tightly managed, to avoid the poor liquidity which could be caused by a long receivables collection
period and comparatively high payables.
The company in the exercise is a service company and hence it would be expected to have low
inventory and a short inventory turnover period. The similarity of receivables collection period and
payables payment period means that the company is passing on most of the delay in receiving
payment to its suppliers.
3 (a) The cash cycle can be found as follows:
Inventory
Inventory turnover period = × 365 days
Cost of sales
plus
Trade receivables
Receivables' days = × 365 days
Sales
less
Trade accounts payables
Accounts payable payment period (payables' days) = × 365 days
Purchases
FINANCIAL ACCOUNTING AND REPORTING | 401

20X2
Total closing inventory ($) 410 000
Cost of goods sold ($) 750 000
Inventory turnover period 199.5 days
Closing receivables ($) 230 000
Sales ($) 900 000

MODULE 6
Receivables collection period 93.3 days
Closing payables ($) 120 000
Purchases ($) 500 000
Payables payment period (87.6 days)
Length of cash cycle (199.5 + 93.3  87.6) 205.2 days

(b) The steps that could be taken to reduce the cash cycle include the following:
(i) Reducing the raw material inventory turnover period.
(ii) Reducing the time taken to produce goods. However, the company must ensure that quality
is not sacrificed as a result of speeding up the production process.
(iii) Increasing the period of credit taken from suppliers. The credit period already seems very
long – the company is allowed three months' credit by its suppliers, and probably could not
be increased. If the credit period is extended then the company may lose discounts for
prompt payment.
(iv) Reducing the finished goods inventory turnover period.
(v) Reducing the receivables collection period. The administrative costs of speeding up debt
collection and the effect on sales of reducing the credit period allowed must be evaluated.
However, the credit period does already seem very long by the standards of most industries.
It may be that generous terms have been allowed to secure large contracts and little will be
able to be done about this in the short term.
4 The total dividend per share is (7.4 + 8.6) = 16 cents
16
× 100 = 5.1%
315
5 (a)
Industry
20X9 20X8 average
ROI 465 + 80 + 15 320 + 60 + 9
= 19.24% = 16.7% 18.5%
3 800 – 890 3 120 – 790
Profit margin 465 + 80 + 15 320 + 60 + 9
(EBIT/Sales) = 5.0% = 4.0% 4.7%
11 200 9 750
Asset turnover 11 200 9 750
= 3.85 × = 4.18 × 3.91 ×
3 800 – 890 3 120 – 790
Current ratio 1 950 1 690
= 2.19  = 2.14  1.90
890 790
Quick ratio 1 230 + 80 1 080 +120
= 1.47  = 1.52  1.27
890 790
Gross profit 11 200 – 8 460 9 750 – 6 825
margin = 24.46% = 30.0% 35.2%
11 200 9 750
Accounts 1 230 1 080
× 365 = 40 days × 365 = 40 days 52 days
receivable
11 200 9 750
collection period
402 | ANALYSIS OF FINANCIAL STATEMENTS

Industry
20X9 20X8 average
Accounts payable 750 690
payment period * × 365 = 32 days × 365 = 37 days 49 days
8 610 6 825
Inventory turnover 8 460 6 825
(times) = 13.22 × = 13.93 18.30 ×
640 490
Gearing 800 + 110 600 + 80
= 30.13% = 22.59% 32.7%
2 110 + 800 + 110 1730 + 600 + 80

* For 20X9 purchases are calculated as (8460 + (640 – 490)). The 20X8 ratio uses cost of sales as
we are unable to calculate the movement in inventory between 20X7 and 20X8.
(b) (i) REPORT
To: Board of directors
From: Accountant Date: XX/XX/XX
Subject: Analysis of performance of DG
This report should be read in conjunction with the appendix attached which shows the
relevant ratios (from part (a)).
Trading and profitability
Return on investment (return on capital employed) has improved considerably between 20X8
and 20X9 and is now higher than the industry average.
Profit after tax as a proportion of sales has also improved noticeably between the years and
is also now marginally ahead of the industry average. Gross margin, however, is considerably
lower than in the previous year and is only some 70 per cent of the industry average. This
suggests either that there has been a change in the cost structure of DG or that there has
been a change in the method of cost allocation between the periods. Either way, this is a
marked change that requires investigation. The company may be in a period of transition as
sales have increased by nearly 15 per cent over the year and it would appear that new non-
current assets have been purchased.
Asset turnover has declined between the periods although the 20X9 figure is in line with the
industry average. This reduction might indicate that the efficiency with which assets are used
has deteriorated or it might indicate that the assets acquired in 20X9 have not yet fully
contributed to the business. A longer term trend would clarify the picture.
(ii) Liquidity and working capital management
The current ratio has improved slightly over the year and is marginally higher than the
industry average. It is also in line with what is generally regarded as satisfactory (2:1).
The quick ratio has declined marginally but is still better than the industry average. This
suggests that DG has no short-term liquidity problems and should have no difficulty in
paying its debts as they become due.
Receivables as a proportion of sales is unchanged from 20X8 and are considerably lower
than the industry average. Consequently, there is probably little opportunity to reduce this
further and there may be pressure in the future from customers to increase the period of
credit given. The period of credit taken from suppliers has fallen from 37 days' purchases to
32 days' and is much lower than the industry average; thus, it may be possible to finance any
additional receivables by negotiating better credit terms from suppliers.
Inventory turnover has fallen slightly and is much slower than the industry average and this
may partly reflect stocking up ahead of a significant increase in sales. Alternatively, there is
some danger that the inventory could contain certain obsolete items that may require
writing off. The relative increase in the level of inventory has been financed by an increased
overdraft which may reduce if the inventory levels can be brought down.
FINANCIAL ACCOUNTING AND REPORTING | 403

The high levels of inventory, overdraft and receivables compared to that of payables
suggests a labour intensive company or one where considerable value is added to bought-in
products.
(iii) Gearing
The level of gearing has increased over the year and is below the industry average. Since the

MODULE 6
return on investment is nearly twice the rate of interest on the loan stock, profitability is likely
to be increased by a modest increase in the level of gearing.
Signed: Accountant
431

ANSWERS TO REVISION
QUESTIONS
FINANCIAL ACCOUNTING AND REPORTING | 433

MODULE 1

1 A The provision of information aimed at running a business more efficiently is an objective of


management accounting rather than financial accounting. The aim of financial reporting is the
provision of information to meet the needs of external users.
2 A Suppliers' interest in their customers' accounts lies in the fact that they wish to be repaid in a
timely fashion. They are therefore interested primarily in the liquidity of a company.
3 D In each case the reverse is correct: US accounting standards are rules-based and IFRS are
principles-based.
4 D The IFRS Foundation is the overseeing body for the IASB, IFRS Interpretations Committee and
IFRS Advisory Council. The Monitoring Board serves as a mechanism for communication
between capital markets authorities and the IFRS Foundation.
5 B The IASB is responsible for issuing IFRS.
6 A The AASB has adopted IFRS equivalent standards (with changes made to reflect the Australian
legislative environment). Although IFRS allows the use of the 'true and fair/fair presentation
override', Australian companies' legislation does not.
7 D The entity's need for additional finance would be shown by its statement of financial position
and information about its investing and financing activities would be shown by its statement of
cash flows (and, to some extent, by its statement of financial position).
8 B Concepts and conventions are contained within the Conceptual Framework, however this is not
the title of a chapter.
9 B If the Conceptual Framework conflicts with an IFRS, the IFRS should be followed. Therefore I is
incorrect.
10 B Its financial position (i.e. its assets and liabilities).
11 A Information about a reporting entity's cash flows during a period helps users assess the entity's
ability to generate future net cash inflows.
12 D A change in accounting policy is applied retrospectively (as required by IAS 8), unless (a) it is
impracticable to do so or (b) the change is required by a new IFRS and the transitional
provisions require/allow prospective application.
A change in accounting policy may be made voluntarily, if the change will result in a more
relevant or reliable presentation in the financial statements.
13 C Users of financial statements are entitled to assume that they have been prepared on the basis
that the entity in question is a going concern, unless there is a clear statement to the contrary.
14 A Accounting standards do not apply to immaterial items.
15 C Verifiability and timeliness are not fundamental characteristics but are two of the four enhancing
qualitative characteristics (the others are comparability and understandability).
16 B A faithful representation is complete, neutral and free from error.
17 B Departure from an IFRS is allowed where compliance would be misleading. Prior agreement
with a regulatory body is not required.
18 A D is the definition of a liability; C is the definition of an asset; B is a mixture of the definitions of
an asset and a liability.
19 A A provision is a liability of uncertain timing and/or amount – the amount is usually based on a
reliable estimate but not necessarily always; it may be known and the uncertainty surrounds timing of
thepayment. Provisions are recognised in the financial statements when certain criteria are met such
as the amount can be measured reliably.
20 D A gain from the sale of a non-current asset is recognised as part of the profit or loss for the
period but the revaluation of a non-current asset is not recognised in profit or loss (it is
recognised in equity as part of other comprehensive income).
21 C The working capital of a business is its current assets and current liabilities.
434 | ANSWERS TO REVISION QUESTIONS

22 A Prepayments are current assets; employee wages are an expense; a revaluation surplus forms
part of equity.
23 A The amount spent on investigating the healing powers of the plant is research rather than
development. At this stage it does not meet the recognition criteria as commercial
development and economic benefit is too distant.
The training costs do not meet the definition of an asset as the resultant benefit is not
controlled by the company (i.e. the trained staff could leave the organisation).
The advertising costs do not meet the definition of an asset as the resultant benefit is not
controlled by the company and there is no certainty of a future economic benefit .
FINANCIAL ACCOUNTING AND REPORTING | 435

MODULE 2

1 D Value in use.
2 A Historical cost is objective in that it is equivalent to the amount paid to obtain an asset. No
estimation nor cost formulae are required.
3 A This is the definition of fair value contained within a number of IFRS.
4 D The first statement relates to normative theory and the second to positive theory.
5 C Operating capital maintenance is based on the productive capacity of an entity, and therefore
requires a maintained level of assets.
6 C Under CPP accounting it is non-monetary items which are restated for the effects of general
price inflation.
7 B A management buy-in is where external managers purchase the company.
8 C The board of directors are responsible for preparing the financial statements (even though the
actual preparation is probably carried out by staff within the finance department with the
assistance of the external auditors).
9 A The auditors' report must be disclosed in the financial statements. It gives an opinion as to
whether the financial statements show a true and fair view and/or are fairly presented.
10 C A statement of financial position and statement of cash flows are required by IAS 1 and IAS 7;
corporate governance disclosures are required as part of compliance with listing rules.
11 B B is not an advantage; competitors may use the information contained within such disclosures in
order to gain advantages in the market.
12 A The details of the members of the various committees are detailed in the corporate governance
report.
436 | ANSWERS TO REVISION QUESTIONS

MODULE 3

1 B Dividends must not be reported in the statement of profit or loss and other comprehensive
income as they are not an expense and therefore do not relate to the performance of an entity
in a reporting period. Dividend payments are reported in the statement of changes in equity
because they represent a transaction between the business and the equity owners in their
capacity as owners.
2 D The credit sale is part of the company's normal operating cycle and so the receivable arising is
classified as current.
The bank overdraft is repayable on demand and so classified as current.
The shares are a current asset investment.
3 D IAS 1 specifies what should be disclosed in the main financial statements.
4 A The revaluation of a property would not be required to be disclosed separately as they would
be automatically included in the ‘total comprehensive income for the year’ figure.
5 B A loss on disposal and depreciation are non cash expenses and so must be added back in the
reconciliation (a profit on disposal is non-cash income which must be deducted).
An increase in payables (or decrease in receivables or inventory) is added back in the
reconciliation (a decrease in payables or increase in receivables or inventory is deducted).
Finance cost is added back to profit before tax in the reconciliation (investment income is
deducted).
6 D $
Profit for the year 12 990
Depreciation 1 300
Purchase of NCAs (6 500)
Increase in receivables (560)
Decrease in inventories 1 100
Increase in payables 230
8 560

7 B $
Carrying amount b/f 90 000
Disposals at carrying amount (3 400)
Purchase of non-current assets (balancing figure) 4 900
Carrying amount c/f 91 500

8 B $
Profit (β) 520 000
Increase in working capital (120 000)
Depreciation 190 000
Current asset investment (800 000)
Loan (230 000)
Share issue 1 400 000
Increase in cash 960 000

9 B $
Issue of shares 17 000
Issue of debentures 70 000
87 000

Dividends received is a cash flow from investing activities.


Interest paid is a cash flow from operating activities.
10 D An external audit can only provide reasonable, not absolute, assurance.
FINANCIAL ACCOUNTING AND REPORTING | 437

MODULE 4

1 B The entity developing the item must be able to sell or use the asset but no formal written
commitment to do so is required.
2 A An intangible asset need not be separable; it must be identifiable.
Development costs must be capitalised if the criteria laid down in IAS 38 are met.
An intangible asset can only be revalued where a fair value is established by reference to an
active market.
3 C Although the production rights are not separable (i.e. capable of separate disposal), they are
contractual and therefore meet the IAS 38 definition of identifiable. The rights must therefore
be recognised as an intangible asset in their own right.
4 D IAS 38 does not require an intangible asset to be amortised where it is assessed to have an
indefinite life. In this case the asset must be tested for impairment annually and whenever there
are indications of impairment.
5 D The research costs should be written off to profit or loss.
Amortisation on the capitalised development costs commences on 1 October, and mirrors the
expected sales pattern:
$
Year 1 68 000
Year 2 68 000
Year 3 68 000
Year 4 68 000
Year 5 34 000
Year 6 34 000
Therefore the total charge to profit or loss is:
$
Research costs 28 000
Amortisation (3/12  68 000) 17 000
45 000

The development costs reported as an asset are therefore $340 000 – $17 000 = $323 000.
6 B
Machine 1 Machine 2
$ $
Cost 450 000 250 000
Depreciation (4/10 and 3/15) (180 000) (50 000)
Carrying amount 270 000 200 000
FV less costs of disposal 285 000 195 000
Value in use 260 000 198 000
Therefore recoverable amount 285 000 198 000
Revised carrying amount 270 000 198 000
Total 270 000 + 198 000 = $468 000
7 B
Carrying amount (900 000  24/25) $864 000
Value in use $860 000
Fair value less costs of disposal ($870 000  95%) $826 500
Recoverable amount $860 000
Impairment loss $4 000

8 A An impairment loss relating to a CGU is initially allocated to any obviously impaired assets.
Corporate assets may be allocated to groups of CGUs where allocation to a single CGU cannot
be achieved on a reasonable and consistent basis. An impairment loss recognised for goodwill
shall not be reversed in a subsequent period.
438 | ANSWERS TO REVISION QUESTIONS

9 D The impairment is allocated first to the goodwill. The remaining $30 000 is split between the
property and machinery on a pro rata basis. Therefore, the machinery is measured at $50 000 –
($30 000  50/250) = $44 000. Note that IAS 36 does not apply to inventories or to financial
assets within the scope of IAS 39; these would include receivables.
10 B Carrying amount of property at 31 December 20X9
$
$600 000  18 / 20 years 540 000
Recoverable amount 535 000
Impairment loss 5 000

As the property has previously been revalued the impairment is charged against the revaluation
surplus and reported as other comprehensive income.
11 C IFRS 15 says any of three criteria must be met for revenue to be recognised over a period of
time. II and III are two of these and the third one is the entity's performance does not create an
asset with an alternative use to the entity and the entity has an enforceable right to payment for
performance completed to date.
12 D Revenue is not recognised on either transaction as Lord has not satisfied the performance
obligations under both contracts.
13 A
$
Customers expected to take up discount (450 000  40%  95%) 171 000
Customers not expected to take up discount (450 000  60%) 270 000
Total 441 000
The sales tax is ignored as this is being collected on behalf of the tax authorities. For the
customers who are expected to take up the discount revenue is recorded net of the settlement
discount as it counts as variable consideration.
14 A
$
Current tax (20X9) 52 300
Over-provision (43 800 – 42 120) (1 680)
Deferred tax (79 320 – 69 780) (9 540)
41 080

15 B
20X9 20X8
Carrying amount 526 260 495 300
Tax written down value 417 600 419 600
Temporary difference 108 660 75 700
 20% 21 732 15 140

Increase in liability = charge to tax $21 732 – $15 140 = $6592


16 A Deferred tax amounts may not be classified as current.
Tax losses are an example of a deductible temporary difference.
17 A A deferred tax asset arises in respect of losses carried forward which can be utilised against
future profits. Losses to be carried forward are ($87 600 – 23 000) = $64 600 giving rise to a
deferred tax asset of ($64 600 x 20%) = $12 920.
18 B The credit balance on the tax account represents the previous year's overprovision. This is
deducted from the current year tax charge ($28 760 – 450) = $28 310.
The current year liability is not adjusted for the over-provision.
FINANCIAL ACCOUNTING AND REPORTING | 439

19 D
Settled Outstanding
payable payable
$ $
Recorded in June (375 000 / 2) / 4.3 43 605 43 605
Settled (375 000 / 2) / 4.6 40 761
Unsettled (375 000 / 2) / 4.5 41 667
Exchange gain 2 844 1 938
Total gain 4 782
20 A
$'000
Assets (560 / 0.8) 700
Share capital (100 / 0.75) 133
Retained earnings (β) 354
Liabilities (170 / 0.8) 213
700

21 D
$ $
Opening net assets at 4.3D / $ 151 163
Opening net assets at 4D / $ 162 500
Gain 11 337
Retained profits at 4.2D / $ 27 381
Retained profits at 4D / $ 28 750
Gain 1 369
Total gain 12 706

22 C The asset is a non-monetary asset and should not be re-translated at the year end (1 450 000 /
6.75) = $214 815; the payable is a monetary amount and must be re-translated (1 450 000 / 7.2) =
$201 388.
23 C Exchange differences arising on settlement of currency items or the retranslation of monetary
items are reported in profit or loss. Exchange differences arising on the translation of financial
statements into the presentation currency are reported as other comprehensive income.
24 C The total amount of lease payment of $26 000 calculated as $2 000 + (3 payments x $8 000) will
be spread over three years to give an annual expense of $8 667.
25 D
$
Deposit 18 000
Present value of future lease payments 150 000
168 000
Depreciation 56 000
168 000 / 3 years
440 | ANSWERS TO REVISION QUESTIONS

MODULE 5

1 B Raleigh Co. cannot have significant influence over Well Co., since Slim Co. already has control
and refuses to listen to Raleigh Co. Therefore, only constitute and investment.
Vic Co. is an associate by virtue of the fact that Raleigh Co. has active representation on
Vic Co.'s board of directors.
2 C Dry Co. controls Wet Co. by virtue of the fact that it directs the relevant activities (operating
activities) of that company.
Dry Co. has significant influence over Cloud Co., evidenced by the 40 per cent shareholding
and participation in the policy-making process.
Dry Co. has significant influence over Drizzle Co., evidenced by the 25 per cent shareholding.
3 C IFRS 10 requires that II, III and IV are all met in order to avoid presenting consolidated financial
statements. I is irrelevant as where subsidiaries operate under long-term restrictions, control
may have been lost. In this case, they are no longer subsidiaries and so consolidated accounts
are not required, as there is no group.
4 D IFRS 10 states that all material subsidiaries should be consolidated; investments in group
companies in individual entity accounts are held at cost or in accordance with IFRS 9; the
accounting policies of the subsidiary must be brought in line with those of the group for the
purposes of consolidation.
5 A Both B and D refer to control.
6 A Car Co.'s accounts should be adjusted as though the cash has been received. Therefore, the
revised balances are:
Receivables $60 000
Overdraft $7 600
7 A
$'000
Consideration 480
Fair value of NCI 45
525
Fair value of net assets (100 + 320 + 30) (450)
75

8 B 20%  (620 000 + 75 000)


9 B
$
Axis Co. 165 000
Yves Co. 180 000
NCA PURP adjustment (see below) (5 478)
Consolidated figure for non-current assets 339 522

Unrealised profit on sale


$25 000 – ($32 000 – ($32 000  20%  2)) 5 800
Reduction in depreciation
($32 000  20%  2 / 12) – ($25 000 / 3yrs  2 / 12) (322)
5 478

Alternatively, the adjustment can be calculated by comparing the carrying amount of the asset at the
reporting date with the carrying amount had no transfer occurred:
Non-current asset post-transfer
$25 000 – ($25 000  2 / 36 months) 23 611
Non-current asset if no transfer had occurred
$32 000 – ($32 000  20%  2 2 / 12yrs) 18 133
NCA PURP 5 478
FINANCIAL ACCOUNTING AND REPORTING | 441

10 B
$
Try Co. 680 900
Ply Co. 80% (532 000 – 400 000) 105 600
Goodwill impairment 25%  68 000 (17 000)
URP 20 / 120  $50 000  ½ (4 167)
Group retained earnings at 31 October 20X9 765 333
11 B
P Co. S Co.  2/12 Adj Group
$'000 $'000 $'000 $'000
Revenue 2 900 300 (20) 3 180
Cost of sales 1 500 150 (20) 1 630
Group gross profit for the year 1 550
ended 31 October 20X9

12 D
An impairment is charged to admin expenses: $'000
Ed Co. 100
Clem Co. 36
Impairment (450 000 – 415 000 – 5000)  20% 6
142

Cost of sales (320 000 + 126 000) 446

13 B 20X9 is not the year of transfer and therefore the URP is made up only of the difference in
depreciation charge:
'Old' depreciation $400 000  10% $40 000
'New' depreciation $340 000 / 8 yrs $42 500
Therefore, an extra $2500 has been charged and must be adjusted for to reduce cost of sales.
14 B 20X9 is the year of transfer and therefore the URP is made up of the loss on transfer and the
difference in depreciation charge:
$
Proceeds 200 000
Carrying amount at transfer (220 000)
Loss on transfer 20 000 to add back to profit

Depreciation:
'old' $300 000 / 15  2 / 12 3 333
'new' $200 000 / 11  2 / 12 3 030
303 extra to charge to profit

Therefore, the overall adjustment is $19 697 to add back to profit.


15 B Dividend income relating to investments other than subsidiaries (and associates) is carried
across to the consolidated statement of profit or loss.
Statement II is correct.
16 B
The associate's revenue is irrelevant to the calculation: $'000
Roulston Co. 1 490
Hudson Co. 430
Roulston sales to Hudson (400)
1 520
17 C
$
Dray Co. 86 500
Ray Co. 73 400
Sales from D to R (40 000)
URP on sales from D to R (40 000  25/125) 8 000
127 900

The URP on the sale to Lay Co. is adjusted against the share of profit of associate.
442 | ANSWERS TO REVISION QUESTIONS

18 C The group share of the URP is adjusted against the investment in the associate:
$
Cost 190 000
Share of post-acquisition profits
$450 000  45% 202 500
URP ($15 000  20%  45%) (1 350)
391 150

19 D Any impairment of an associate is charged against the profits of the associate.


20 C This is a parent associate relationship; significant influence is evidenced by the material
transactions between the two companies.
$
Cost of investment 160 000
Group share of post-acquisition profits
16% × $98 000 15 680
175 680

There is no unrealised profit, because all the goods have been sold on outside the group.
FINANCIAL ACCOUNTING AND REPORTING | 443

MODULE 6

1 B Interest cover is an indicator of solvency; dividend cover is an investor ratio.


2 D ROI is the product of profit margin and asset turnover. An increase in either of these will
increase ROI. Interest cover and the current ratio are irrelevant to ROI.
3 A A decrease in expenses increases profits and so equity and decreases the gearing ratio.
A decrease in the allowance for receivables also increases equity.
An upward revaluation increases equity and so decreases the gearing ratio.
The payment of a dividend decreases equity and so increases the gearing ratio.
4 B The quick ratio of 20X8 was:
12 840 + 1 880
= 0.8
17 200 + 1 200
Therefore in 20X9:
? + 1 280 + 348
= 0.8
10 760
? + 1 628 = 0.8  10 760
? + 1 628 = 8 608
Receivables = 6 980
5 A Earnings per share is calculated as the profits attributable to the ordinary shareholders divided
by the number of shares. The profits attributable to ordinary shareholders is profit after tax and
therefore:
1 870 000
= 0.0748
No. of shares
1 870 000
No. of shares = = 25m
0.0748
6 C This is correct because ratios are only useful if they can be compared to other relevant and
comparable information so relative performance and position can be considered.
The other statements are incorrect for the following reasons:
Ratios are a useful tool to all stakeholders (including management) not just investors. Ratios
based on past performance are not necessarily indicative of what will happen in the future - for
example, the general economy might change, a new competitor might enter the market, an
entity might introduce a new product.
7 A The correct answer is: 24%
Earnings before interest and tax
Return on investment = × 100%
Total assets less current liabilities
= (10,200/42,500) × 100%
= 24%
8 C Net profit margin will decrease because as the assets are revalued upwards, the depreciation
will increase making profit lower and the profit margin decrease accordingly assuming all else
stays the same.
A revaluation will have no effect on the current ratio.
An upwards revaluation of property, plant and equipment and will make gearing ratio
(debt/equity) decrease as equity has increased.
Interest cover is likely to decrease because profit before interest and tax will fall due to the
higher depreciation.
Non-current asset turnover is calculated as revenue/non-current assets, therefore, it too will
decrease.
444 | ANSWERS TO REVISION QUESTIONS

9 B Gearing and interest cover ratios.


Although all these ratios could be of interest to the bank, as a long-term loan creditor the bank
will be particularly interested in factors that affect the company's ability to pay interest. These
include: whether it is getting into further debt or improving its situation (improving gearing ratio
by lowering the gearing ratio); and the level of profit in relation to interest expense.
10 A GF won a new contract with the largest network provider and retailer in the Middle East on the
basis of a bulk buy discount.
The operating margin in the Middle East has fallen in 20X7 (10.5%) from 20X6 (11.5%). However,
revenue has increased by 56.5%. A new contract could explain the reason for the increase in
revenue. The fact that the customer has negotiated a bulk buy discount could explain the
reason for the decline in operating margin as it implies a lower selling price that normal which
would cause the operating margin to decrease.
The other answers are incorrect because:
An increase in demand for mobile phones would cause revenue to increase but on its own
would not cause a change in operating margin unless there is also a change in sales price, a
change in production costs or a change in administrative and distribution expenses.
If GF had only just begun selling phones in the Middle East, you would expect the margin to be
lower in the early years due to start-up costs and then increasing year on year. However, here,
the operating margin has decreased in 20X7.
High-end smartphones would be likely to attract a higher margin than other devices which
would result in an increase rather than a decrease in operating margin.

You might also like